Sappppppp 1

You might also like

Download as docx, pdf, or txt
Download as docx, pdf, or txt
You are on page 1of 127

DIVISION LAS DEVELOPMENT TEAM FOR

SENIOR HIGH SCHOOL – STATISTICS AND PROBABILITY

Norma B. Samantela, CESO VI Schools Division Superintendent


Ma. Jeany T. Abayon, Ed.D. Assistant Schools Division Superintendent
Fatima D. Buen, CESO VI Assistant Schools Division Superintendent

DIVISION EVALUATION TEAM


CID Chief, Dr. Sancita B. Peñarubia
PSDS/Division Mathematics Coordinator, Dr. Felma A. Bonito

WRITERS:

Quarter 3 Quarter 4

Catherine R. Realingo Catherine R. Realingo

Marijo S. Balin Marijo S. Balin

Amie-Lyn S. Cedo Amie-Lyn S. Cedo

Charmie G. Rellores Charmie G. Rellores

Aris B. Nisola Aris B. Nisola

Margie C. Macatingrao Margie C. Macatingrao

Giovanni G. Naag Giovanni G. Naag

Albert S. Corbilla Albert S. Corbilla

Hilda C. Remendado Hilda C. Remendado

Mary Francia S. Rico Mary Francia S. Rico

Team Leader : Elenita L. Nocidal


Content Editors : Elenita L. Nocidal
Jocelyn P. Castelo
Cover Layout Artist : Albert S. Corbilla
IT : Joseph C. Bueno

CONTENT

Week No. Learning Competencies Page No.


Illustrate a random variable (M11/12SP-IIIa-1)
Distinguish between a discrete and a continuous
random variable (M11/12SP-IIIa-2) 1
Week 1 Find the possible values of a random variable
(M11/12SP-IIIa-3)
Illustrates a probability distribution for a discrete
random variable and its properties. M11/12SP-IIIa-4
Constructs the probability mass function of a
discrete random variable and its
6
corresponding histogram. M11/12SP-IIIa-5

Week 2
Computes probabilities corresponding to a given
random variable. M11/12SP-IIIa-6
Illustrates the mean of a discrete random variable.
M11/12SP-IIIb-1
Calculate the mean of a discrete random variable.
M11/12SP-IIIb-2
14
Interprets the mean of a discrete random variable.
M11/12SP-IIIb-3

Solves problem involving mean of a discrete


probability distribution. M11/12SP-IIIb-4
Calculate the Variance of a Discrete Random
Variable (M11/12SP-111b-2)

Interprets the Variance of a Discrete Random 19


Variable (M11/12SP-111b-3)
Week 3
Solve problems involving Variance of probability
distribution (M11/12SP-IIIb-4)
Illustrate normal random variable and its
26
characteristics (M11/12SP-IIIc-1)
Identifies region under the normal curve
Week 4 corresponding to different standard normal vales. 32
M11/12SP-111C3

Convert a normal random variable to a standard


40
normal variable and vice versa (MII/12SP-IIIc4)
Compute probabilities and percentiles using
standard normal table (MII/12SP-IIIc-d-1) 46

The learner illustrates random sampling.


51
(M11/12SP-IIIc-2)
The learner distinguishes between parameter and
Week 5 57
statistic. (M11/12SP-IIIc-3)
The learner identifies sampling distributions of
62
statistics (sample mean) – M11/12SP-IIIc-
Find the Mean and Variance of the Sampling
Distribution of the Sample Mean (M11/12SP-IIId-5)

Week 6 Define the sampling distribution of the sample mean 69


for normal population when the variance is (a)
known; and (b) unknown (M11/12SP-IIIe-1)

Illustrates the Central Limit Theorem: M11/12SP-


IIIe-2
Defines the sampling distribution of the sample 80
Week 7 to 8 mean using the Central Limit Theorem: M11/12SP-
III-3
Solve problems involving sampling distributions of
86
the sample mean. M11SP/IIIe-f-1
The learner illustrates t- distribution (M11/12SP-
91
IIIg-2)
Week 9
Identifying percentiles using t-distribution
97
(M11/12SP-IIIg-5)
Identifies the length of a confidence interval: LC
Code: M11/12SP-IIIj-1
Week 10 99
Computes for the length of confidence interval: LC
Code: M11/12SP-IIIj-1
Computes for an appropriate sample size using the
length of the confidence interval. LC Code:
M11/12SP-IIIj-3
106
Solve problems involving sample size determination.
LC Code: M11/12SP-IIIj-4
STATISTICS AND PROBABILITY
Name of Learner: _____________________________ Section: _________________ Grade
Level: _________________________________ Date: ___________________

LEARNING ACTIVITY SHEET


EXPLORING RANDOM VARIABLE

I. BACKGROUND
You have learned in your past lessons in mathematics that an experiment is any activity, which
can be done repeatedly under similar conditions. The set of all possible outcomes of an experiment is
called the sample space. You have also learned how to systematically list the possible outcomes of a
given experiment.
Let us recall, the finding the sample space of an experiment.
• In throwing a die once, the sample space is S = {1, 2, 3, 4, 5, 6}.
• The sample space of rolling a die and tossing a coin simultaneously is,

S=1H,1T,2H,2T,3H,3T,4H,4T,5H,5T,6H,6T
• The sample space of tossing three coins is
S= {TTT, TTH, THT, HTT, HHT, HTH, THH, HHH}.
This time this activity sheet will explain to us the concept of random variables, find the possible
values of it and its classification.
A random variable is a set of possible values for a random experiment. It is denoted by a
capital letter, usually X, Y and Z.
In some random experiments such as:
• Tossing coin three times
• Rolling a dice twice;
• Drawing two balls in a box
To illustrate and give the value of random variables using the following problems,
Example 1: In tossing a coin, let X be the random variable denoting the number of tails. Find
the value of the random variable X.
Random variable Possible Outcomes Possible values

HEAD O

X = no. of tails TAIL 1


So, the possible values of the random variable X are 0 and 1.

Example: 2. A coin is tossed 3 times. Let X be the random variable denoting the number of heads.
Find the values of the random variable X .

Solution: To do this, identify all the possible outcomes of the given statistical experiment.
Determine the specific random variable defined in the problem i.e.
Outcomes TTT TTH THT THH HTT HTH HHT HHH

𝑥 = 𝑛𝑜. 𝑜𝑓 ℎ𝑒𝑎𝑑𝑠 0 1 1 2 1 2 2 3

1
Therefore, the possible values of the random variable X are 0, 1, 2 & 3. Example:
3.The number of even number outcomes in a roll of a die. Let Y be the random variable
denoting the number of even numbers .Write the possible values of Y.
Solution: Outcomes = since we are looking for even
number, therefore, the possible values of the random variable X
are , & .

Types of random variables


• Discrete random variable is a variable whose value is obtained by counting. Examples:
1. The number of students in a class
2. The number of test questions answered correctly.
3. The results or outcomes of rolling 2 dice.

• Continuous random variable is a variable whose value is obtained by measuring.


Examples:
1. The amount of time required to complete a project.
2. The amount of rain, in inches, falls in a storm.
3. The height of children
4. The amount of times it takes to sell shoes

II. Learning Competencies:

• Illustrate a random variable (M11/12SP-IIIa-1)


• Distinguish between a discrete and a continuous random variable (M11/12SP-IIIa-2)
• Find the possible values of a random variable (M11/12SP-IIIa-3)

III. Exercises/Activity

Activity 1

CLASSIFY ME

Directions: Classify the following random variables as discrete or continuous.


____________ 1. The number of defective computers produced by a manufacturer in a
certain school
____________ 2. The weight of new born each year in a hospital
____________ 3. The number of siblings in a family of a region
____________ 4. The amount of paint utilized in a classroom building project of BNHS
____________ 5. The number of dropout in a school district for a period of 10 years
____________ 6. The speed of a car
____________ 7. The numbers of female athletes of the Sports track
____________ 8. The time needed to finish the test
____________ 9. The amount of sugar in a cup of coffee
____________10. The number of people who are playing LOTTO each day

2
THINK & REFLECT:
How do you know whether a random variable is continuous or discrete?
______________________________________________________________________________ What
is the difference between continuous and discrete random variables?
_______________________________________________________________________________

Activity 2

RE-VISIT & RECALL

Directions:
List the sample space of the following experiments
Experiment Sample Space

1. Tossing two coins.

2. If a pair of dice rolled which the sum is 5.

3. Drawing a diamond from a deck of cards.

4. Getting a defective item when two items are randomly


selected from a box of two defective and three non-defective
items.

THINK & REFLECT:


How could you determine the possible outcomes/sample space of the following experiment?
___________________________________________________________________________

Activity 3

SOLVE ME

Directions: Illustrate the problem using table values and solve.

3
1. Suppose three cell phones are tested at random where D represent the defective cell phone and N
represent the non-defective cell phone. Let X represents the number of defective cell phones that
occur. , show the values of the random variable X .
2. Two balls are drawn in succession without replacement from an urn containing 5 red balls and 6
blue balls. Let Z be the random variable representing the number of blue balls. Find the values of
the random variable Z .
3. Four coins are tossed. Let Y be the random variable representing the number of heads that occur.
Find the values of the random variable Y .
4. A shipment of five computers contains that are slightly defective. If a retailer receives three of
these computers at random, list the elements of the sample space S using the letters D and N for
defective and non-defective computers, respectively. To each sample point, assign a value x of the
random variable X representing the number of computers purchased by the retailer which are
slightly defective.
5. A family has four children. If Y is a random variable that pertains to the number of female
children, what are the possible values of Y?

THINK & REFLECT:


How do you find the value of random variables?
___________________________________________________________________________
__________________________________________________________________________________
__________________________________________________________________________________
Enumerate the steps.
___________________________________________________________________________
__________________________________________________________________________________
_________________________________________________________________________________

4
References:
Statistics and Probability-first edition Probability Distribution of Random Variables (Modules
Author: Rene R. Belecina, Elisa S. Baccay Author: Daisy Rodriquez Acas
Efren B. Mateo
Statistics & Probability (Grade 11)Q3– Module 1: Random Variable & Probability Distributions
First Edition, 2020 Authors: Monina C. Raagas,Mardy Nelle V. Galve

5
Prepared by:

CATHERINE R.REALINGO
BONBON NATIONAL HIGH SCHOOL

STATISTICS AND PROBABILITY


6
Name: ____________________________________ Grade Level: _________________________
Section: ___________________________________ Date: _______________________________

LEARNING ACTIVITY SHEET #2


CONSTRUCTING PROBABILITY DISTRIBUTIONS

Welcome back dear young mathematician! You are now on second week of Quarter
3. In the previous week you were able to illustrate a random variable, classify them as
discrete or continuous; and find the possible values of it.

This week, we will be exploring Probability Distributions of Discrete Random


Variable . After knowing the possible outcomes, you can compute for the corresponding
probabilities, construct probability distribution and draw a histogram (bar graph).

Learning this topic will probably help you in decision-making.


Decisionmaking is an important aspect in business, education, insurance, other real -
life

situations. Many decisions are made by assigning probabilities to all possible

outcomes pertaining to the situation and then evaluating the results. For

instance, an insurance company might be able to assign probabilities to the number


of vehicles a family owns. This information will help the company in
making decisions regarding future financial situations. This situation requires the use of random
variables and probability distribution.

I. Background Information for Learners

Discrete Probability Distribution


A Discrete Probability Distribution or a Probability Mass Function consists of the values a
random variable can assume and the corresponding probabilities of the values.

Examples 1.
Number of Heads

Suppose three coins are tossed. Let Y be the random variable representing the
number of heads that occur. Find the probability of each of the values of the random variable Y.

Solution : The sample space for this experiment is : S= {TTT,TTH,THT,HTT,HHT,HTH,THH,HHH}


H- Head T-Tail

7
Possible Outcomes TTT TTH THT HTT HHT HTH THH HHH
Values of the 0 1 1 1 2 2 2 3
Random Variable Y
There are four possible values of the random variable Y representing the number of heads. These are 0,1,2,
and 3.
What to Do Next ? Assign probability values P(Y) to each value of the random variable and make a table.
You can write it in vertical/horizontal way.

Remember! Since there are 8 possible outcomes, and out of these, 1 outcome got no heads(TTT), therefore
the probability of getting 0 heads is 1/8. Do these also to other possible outcomes.

Number of Heads Y 0 12 3
Probability P(Y)

Table 1.1. The Probability Distribution or the Probability Mass Function of Discrete Random Variable Y

Examples 2.
Number of Blue Balls

Two balls are drawn in succession without replacement from an urn containing 5 red balls
and 6 blue balls. Let Z be the random variable representing the number of blue balls. Construct the
probability distribution of the random variable Z.

Solution : The sample space for this experiment is : S= {RR,RB,BR,BB} R- Red B-Blue

Possible Outcomes Value of Random Variable Z


(Number of Blue Balls)
RR 0
RB 1
BR 1
BB 2

There are three possible values of the random variable Z representing the number of blue balls. These
are 0,1,and 2.

Number of Blue Balls Z 0 1 2

2
Probability P(Z) 4 or
8
Table 1.2. The Probability Distribution or the Probability Mass Function of Discrete Random Variable Z

Can you make a histogram for this probability distribution ?

Histogram for the Probability Distribution of Discrete Random Variable

Remember, a histogram is a bar graph. To construct a histogram for a probability distribution,

• Plot the values of the random variable (X, Y, etc.) along the horizontal axis.
• Plot the probabilities (P(X)) along the vertical axis.
Extra Tips :
1. If you are dealing with fractions, you can convert fractions to decimals for easy plotting.
2. In making the axes, make sure that you have equal intervals/spacing with the numbers.

Remember, ¼ is equivalent to 0.25 and ½ is equal to 0.50.

0.6
Probabilities P(Z)

0.5
0.4
0.3
0.2
0.1
0
0 1 2
Number of Blue Balls (Z)

Figure 1.1 The Histogram for the Probability Distribution of the Discrete Random Variable Z

Examples 3.
Number of Defective Cell Phones

Suppose three cell phones are tested at random. Let D represent the defective cell phone and let N
represent the non-defective cell phone. If we let X be the random variable for the number of
defective cell phoned, construct the probability distribution of the random variable X.

Solution :
Possible Outcomes Value of the Random Variable X
(number of defective cell phones)
NNN 0
NND 1
NDN 1
DNN 1
NDD 2
DND 2
DDN 2

9
DDD 3

Table 1.3 Probability Distribution or Probability Mass function of Discrete Random Variable X.
Number of Defective Probability P(X)
Cell phones X
0

0.4
0.35
Probabilities P(X)

0.3
0.25
0.2
0.15
0.1
0.05
0
0 1 2 3
Number of Defective Cell Phones (X)

Figure 1.2 The Histogram for the Probability Distribution of the Discrete Random Variable Z

Properties of a Probability Distribution

1. The probability of each value of random variable must be between or equal to 0 and 1. In symbol, 0 ≤ 𝑃(𝑋) ≤
1.

2. The sum of the probabilities of a value is equal to 1. In symbol, ∑ 𝑃(𝑋) = 1 .

Examples 4.
Daily Demand for Copies of a Movie Magazine

The daily demand for copies of a movie magazine at a variety store has the probability distribution as
follows :

Number of Copies X Probability P(X)

10
0 0.06
1 0.14
2 0.16
3 0.14
4 0.12
5 0.10
6 0.08
7 0.07
8 0.06
9 0.04
10 0.03

a. What is the probability that three or more copies will be demanded in a particular day?
b. What is the probability that the demand will be at least two but not more than six? Solution :
a. 𝑃(𝑋 ≥ 3) = 0.14 + 0.12 + 0.10 + 0.08 + 0.07 + 0.06 + 0.04 + 0.03
𝑃(𝑋 ≥ 3) = 0.64

The probability that three or more copies will be demanded in a particular day is 0.64

b. 𝑃(2 ≤ 𝑋 ≤ 6) = 0.16 + 0.14 + 0.12 + 0.10 + 0.08


𝑃(2 ≤ 𝑋 ≤ 6) = 0.60
The probability that the demand will be at least two but not more than six is 0.60.
II. Learning Competencies :
• Illustrates a probability distribution for a discrete random variable and its properties. M11/12SP-IIIa-4
• Constructs the probability mass function of a discrete random variable and its corresponding
histogram. M11/12SP-IIIa-5
• Computes probabilities corresponding to a given random variable. M11/12SP-IIIa-6

III. Exercises/ Activities

Activity #1 – YES or NO ?
Directions. Determine whether the distribution represents a probability distribution. Write YES if it is
probability distribution. Otherwise, write NO. Explain your answer.
1.

2.

3.

5. 𝑃(1) = ,P (2) = , 𝑃(3) =


11
Activity #2 – CONSTRUCT ME!

Directions. Construct the probability distribution for the random variables described in each of the
following situations. Draw the corresponding histogram for each probability distribution. Label it
properly.
1. Let T be a random variable giving the number of tails in three tosses of a coin. List the elements
of the sample space S for the three tosses of the coin. And assign a value to each sample point.
Value of the Random Variable T Probability P(T)
(Number of Tails)

2. The probabilities P(B) that a student will borrow 1,2,3,or 4 books are 0.45, 0.30,0.15, and 0.10,
respectively.

Activity #3 – FIND ME

Directions. Based from the situation and table below, find the probabilities. Show your solutions.

The following data show the probabilities for the number of cars sold in a given day at a car dealer store.

Number of Cars X Probability P(X)


0 0.100
1 0.150
2 0.250
3 0.140
4 0.090
5 0.080
6 0.060
7 0.050
8 0.040
9 0.025
10 0.015

3. Find 𝑃(𝑋 ≤ 2).

12
4. Find 𝑃(𝑋 ≥ 7).
5. Find 𝑃(1 ≤ 𝑋 ≤ 5).

IV. Questions to Ponder


• How do you construct probability distribution ?
_____________________________________________________________________
_____________________________________________________________________

• How do you make a histogram for a probability distribution? Give the steps in constructing
a histogram for probability distribution.
_____________________________________________________________________
_____________________________________________________________________

V. Reflection
1. What are your learnings from the activities ?
___________________________________________________________________________

2. What did you like most from this lesson ?


__________________________________________________________________________ .

3. What is the most challenging or least interesting from the activities?


_________________________________________________________________________ .

VI. Reference for Learners


Belecina, Rene R., etal. (2016). Statistics and Probability First Edition, REX Book Store,
Inc., Florentino St., Sta . Mesa Heights, Quezon City, Philippines, pp. 9 – 20.

13
14
V Answ
II.er
Activi–YES
Key
ty #1 or
1 . NO, because the sum of the1.
NO?
.
2 probabilities P(X) is 5/3 which not
3 . NO, because the sum of the 1.
YE
equal to
4 probabilities
.S because the sum
P(X) of the
is 0.94 probabilities
which is not
5 .NOP(X)tois 1.1 which is not equal to 1.
equal
YE
,
Activ
S –CONSTR
ity UCT ME!
1
#2
. Value of the Probabi
( Number
Random ) lity P(T)
of H
Variable
TailsT 0
H 1
H 1
T 1
H 2
T
H 2
TH 2
T 3
T
H
T
Tab4Probability Distribution or Probability
T.
le MassNum Tai
function ofProba
Discrete
T) Random
ber0ls
1. Variable 1
bility
of T P( 8
1 1
8
2 1
8
3 1
8
15
Prepared by :

possible
then probability will occur.
“The first step is to establish that something is
it!
Congratulations young mathematician! You made
2.
Table Probability Distribution or Probability Mass function of Discrete Random
1.5 Value
Variable
of theB.Random Variable Probability
B P(B)
( Number of Books)
1 0.4
5
2 0.3
0
3 0.1
5
4 0.1
0
FigureThe Histogram for the Probability Distribution of the Discrete Random
1 Variable B
Activity –#3FIND
ME
1 . �(� ≤2) = 0.100 + 0.150 +
��(��≤�) =0.250
� � � 0.500
= 2 . �(�≥7) 0.050 + 0.040 + 0.025 + 0.015
��(��≥�) =
� � � 0.13
MARIJO S. BALIN
Writer
STATISTICS AND PROBABILITY
Name : ____________________________________ Grade Level : _________________________
Section : ___________________________________ Date : _______________________________

LEARNING ACTIVITY SHEET #3


COMPUTING THE MEAN OF A DISCRETE PROBABILITY DISTRIBUTION

Hello dear young statistician! You are now on second wee k of Quarter 3. In the
previous lesson you were able to construct probability distribution of discrete random variable
and make a histogram.
This week, we will be exploring Mean of Probability Distribution. You
have learned from the previous
study of mathematics how to find the mean of

ungrouped and grouped data. You also learned how to describe data using mean.

In this lesson , you will learn how to compute the mean of a discrete probability
distribution. You will also learn how to interpret the mean of a discrete probability
distribution.

III. Background Information for Learners

Mean of the Random Variable X or the Mean of the Probability Distribution of X


➢ The mean tells us the average number/ values that would appear.

Formula for the Mean of the Probability Distribution

The mean of a random variable with a discrete probability distribution is


𝜇 = 𝑋1 ∙ 𝑃(𝑋1) + 𝑋2 ∙ 𝑃(𝑋2) + ⋯ + 𝑋𝑛 ∙ 𝑃(𝑋𝑛) or
𝜇 = ∑ 𝑋 ∙ 𝑃(𝑋)
where: 𝑋1, 𝑋2,𝑋3,…𝑋𝑛 are the values of the random
variable X; and 𝑃(𝑋1), 𝑃(𝑋2),𝑃(𝑋3)…𝑃(𝑋𝑛) are the
corresponding probabilities
Steps Solution
1. Construct the probability distribution Number of Spots Probability P(X)
for the random variable X X
representing the number of spots 1
that would appear.
2

4
16
5

6
Example 1
Number of Spots
Consider rolling a die. What is the average number of spots that would appear?
2. Multiply the value of the random Number of Probability 𝑋 ∙ 𝑃(𝑋) Solution:
variable X by the corresponding Spots X P(X)
probability. 1

3. Add the results obtained in Step 2

The mean of the random variable X or the mean of the probability distribution of X is 3.5
This implies that the average number of spots that would appear in a roll of a die is 3.5.

Example 2
Grocery Items

The probabilities that a customer will buy 1,2,3,4, or 5 items in a grocery are , , , , 𝑎𝑛𝑑
, respectively.What is the average number of items that a customer will buy?

Solution:

Number of Items X Probability P(X) 𝑋 ∙ 𝑃(𝑋)


1

17
2

The mean of the probability distribution of X is 3.1


This implies that the average number of items that the customer will buy is 3.1 or theoretically 3 items.

Example 3
Surgery Patients

The probabilities that a doctor operates on 3,4,5,6, or 7


patients in any day are 0.15, 0.10, 0.20,0.25, and 0.30, respectively.
Find the average number of patients that a surgeon operates on a
day.

Solution:
Number of Patients X Probability P(X) 𝑋 ∙ 𝑃(𝑋)
3 0.15 0.45
4 0.10 0.40
5 0.20 1.00
6 0.25 1.50
7 0.30 2.10
∑ 𝑋 ∙ 𝑃(𝑋) = 5.45
The average number of patients that a surgeon will operate in a day is 5.45.

IV. Learning Competencies :


• Illustrates the mean of a discrete random variable. M11/12SP-IIIb-1
• Calculate the mean of a discrete random variable. M11/12SP-IIIb-2
• Interprets the mean of a discrete random variable. M11/12SP-IIIb-3
• Solves problem involving mean of a discrete probability distribution. M11/12SP-IIIb-4

III. Exercises/ Activities

Activity #1 – COMPLETE ME!

18
Directions. Complete the table below and find the mean of the probability distribution.
1.
X P(X) 𝑋 ∙ 𝑃(𝑋)
1

11

16

21

∑ 𝑋 ∙ 𝑃(𝑋) =
The mean of the probability distribution is ______.
2.
X P(X) 𝑋 ∙ 𝑃(𝑋)
3 0.15
6 0.35
8 0.40
10 0.10
∑ 𝑋 ∙ 𝑃(𝑋) =

The mean of the probability distribution is ______.

Activity #2 – WHAT DO YOU MEAN ?


Directions. Make a table for the situation below and find the mean of the probability distribution. Show
your complete solution.

The probabilities of a manufacturing 0,1,2,3,4, or 5 defective parts in one day are 0.75,
0.17, 0.04,0.025,0.01, and 0.005, respectively. Find the mean of the probability distribution.

Activity #3 – IS IT BELIEVABLE ?
Directions. Read the situation below, show your full solution and give your explanation briefly.

A bakeshop owner determines the number of boxes of pandesal that are delivered
each day. Find the mean of the probability distribution shown. If the manager stated that 35
boxes of pandesal were delivered in one day, do you think that this a believable claim? Why?

Number of Boxes (X) Probability P(X)


35 0.10
36 0.20
37 0.30
38 0.30
39 0.10

19
IV. Questions to Ponder
• What are the steps in computing the mean of a probability distribution of a random
variable? How is it similar to the procedure in finding the mean of a frequency
distribution?
_____________________________________________________________________
_____________________________________________________________________
_____________________________________________________________________

• Compare the formulas for finding the mean of a frequency distribution ( 𝑋 = ∑ 𝑓𝑥𝑚
)
𝑛 and that of a probability distribution ∑ 𝑋 ∙ 𝑃(𝑋).
_____________________________________________________________________
_____________________________________________________________________
_____________________________________________________________________
V. Reflection

Draw what you feel after doing the different activities on Computing the Mean of a Discrete Probability
Distribution and explain it.

VI. References for Learners


Belecina, Rene R., etal. (2016). Statistics and Probability First Edition, REX Book Store, Inc., Florentino
St., Sta . Mesa Heights, Quezon City, Philippines, pp. 21 – 30.

20
Prepared by :

MARIJO S. BALIN
21
Writer
Name: Grade
Level:__________________________
Section: ________________________________ Date: _______________________________

LEARNING ACTIVITY SHEET


INTERPRETING THE VARIANCE OF A DISCRETE RANDOM VARIABLE

I. Background Information for Learners


This activity sheet serves a guide to the learners. This will focus in the learner’s mastery on interpreting the
variance of a discrete random variable.
You’ve learned from the previous Learning Activity Sheet how to find the Mean of a Discrete
Random Variable. This time, let us find the Variance of a discrete random variable, Interpret the
Variance of a discrete random variable and Solve problems involving the variance of probability
distribution.
We have seen that the mean of a random variable Χ is a measure of the central location of the
distribution of Χ. If we are summarizing features of the distribution of Χ, it is clear that location is not
the only relevant feature. The second most important feature is the spread of the distribution.
If values of Χ near its mean 𝜇x are very likely and values further away from 𝜇x have very small
probability, then the distribution of Χ will be closely concentrated around 𝜇x. In this case, the spread
of the distribution of Χ is small. On the other hand, if values of Χ some distance from its mean 𝜇x are
likely, the spread of the distribution of Χ will be large.
These idea leads to the most important measure of spread, the variance and a closely related measure, the
standard deviation.
For a start, here are the steps on how to find the Variance of a Discrete Random Variable. This will
serve as your guide as you do the activity later on.

Steps in Finding the Variance and Standard Deviation 1.


Find the mean of the probability distribution.
2. Subtract the mean from each value of the random variable X.
3. Square the results obtained in Step 2.
4. Multiply the results obtained in Step 3 by the corresponding probability.

5. Get the sum of the results obtained in Step 4.

Formula for the Variance and Standard Deviation of a Discrete

Probability Distribution
The variance of a discrete probability distribution is given by the formula:
𝝈𝟐 = ∑(𝑿 − 𝝁)𝟐 ∙ 𝑷(𝑿)
The standard deviation of a discrete probability distribution is given by the formula:

𝝈 = √∑(X − μ)² ∙ P(X)

Where:
X = value of the random variable
22
P(X) = probability of the random variable X
𝜇 = mean of the probability distribution

II. Learning Competency:


1. Calculate the Variance of a Discrete Random Variable ( M11/12SP-111b-2)
2. Interprets the Variance of a Discrete Random Variable ( M11/12SP-111b-3)
3. Solve problems involving Variance of probability distribution (M11/12SP-IIIb-4)

III. Activity:

A. Number of Almonds
The random variable X, representing the number of almonds in a Fruit and Nut chocolate bar has the
following probability distribution. Compute the variance.

X 1 2 3 4 5

P (X)

Step 1: Find the mean of the probability distribution using the formula 𝜇 =
Number of Almonds Probability

X P (X) 𝑿
1 𝟐
1• =
𝟏𝟎
2 𝟔
2• =
𝟏𝟎
3 𝟗
3• =
𝟏𝟎
4 𝟒
4• =
𝟏𝟎
5 𝟓
5• =
𝟏𝟎
𝜇 � �
𝟐𝟔
𝟏𝟎

Step 2. Subtract the mean from each value of the random variable X.
Number of Almonds Probability P(X) 𝑿
X –𝜇
𝑿
1 .𝟔

23
2 .𝟔

3 .𝟒

4 .𝟒

5 .𝟒

Step 3. Square the results obtained in Step 2.


Number of Almonds Probability
X P(X)
𝑿 𝑿–𝝁 (𝑿 – 𝝁)²

1 −1.6 (−1.6)² = 2.56

2 −0.6 (−0.6)² = 0.36

3 0.4 (0.4)² = 0.16

4 1.4 (1.4)² = 1.96

5 2.4 (2.4)² = 5.76

𝜇 2.6

Step 4. Multiply the results obtained in Step 3 by the corresponding probability.


Number of Probability
Almonds P(X)
X 𝑿 𝑿–𝜇 (𝑿 – 𝜇)² (𝑋 − 𝜇)² • 𝑷(𝑿)

1 −1.6 2.56
2.56 • = 0.512
2 −0.6 0.36
0.36 • = 0.108
3 0.4 0.16
0.16 • = 0.048
4 1.4 1.96
1.96 • = 0.196
5 2.4 5.76
5.76 • = 0.576
𝜇 2.6 )=
1.44

Step 5. Get the square root of the variance to get the standard deviation.
The variance 𝜎2 of the probability distribution is 1.44
The standard deviation is 𝜎
What does it mean?
The variance (1.44) is the weighted average of the squared deviation from the mean (2.6), where the
weights are given by the probability function.

24
B. Number of Face Mask
The number of boxes of face mask sold per day at a drugstore, with its corresponding probabilities, is shown
in the table. Find the variance and standard deviation of the probability distribution.

Number of boxes sold Probability P


X (X)
19 0.20
20 0.20
21 0.30
22 0.20
23 0.10

Step 1: Find the mean of the probability distribution using the formula 𝜇 ∑ ∙ ( ).
Number of Almonds Probability

X P (X) 𝑿

19 0.20 19•0.20 = 3.8

20 0.20 20•0.20 = 4.0

21 0.30 21•0.30 = 6.3

22 0.20 22•0.20 = 4.4

23 0.10 23•0.10 = 2.3

𝜇 = ∑ ∙ ( ) = 20.8

Step 2. Subtract the mean from each value of the random variable X.
Number of Almonds Probability

X P (X) 𝑿 𝑿−𝜇

19 0.20 3.8 19 – 20.8 = -1.8

20 0.20 4.0 20 – 20.8 = -0.8

21 0.30 6.3 21 – 20.8 = 0.2

22 0.20 4.4 22 – 20.8 = 1.2

23 0.10 2.3 23 – 20.8 = 2.2

𝜇 =∑ ∙ ( )
20.8

Step 3. Square the results obtained in Step 2

25
Number of Probability
Almonds
P (X) 𝑿 𝑿−𝜇 (𝑿 – 𝜇)²
X

19 0.20 3.8 -1.8 (-1.8)² = 3.24

20 0.20 4.0 -0.8 (-0.8)² = 0.64

21 0.30 6.3 0.2 (0.2)² = 0.04

22 0.20 4.4 1.2 (1.2)² = 1.44

23 0.10 2.3 2.2 (2.2)² = 4.84

𝜇 = ∑ ∙ ( ) = 20.8

Step 4. Multiply the results obtained in Step 3 by the corresponding probability.


Number of Probability
Almonds
P (X) 𝑿 𝑿−𝜇 (𝑿 – 𝜇)² (𝑋 − 𝜇)² • 𝑷(𝑿)
X

19 0.20 3.8 -1.8 3.24 3.24 • 0.20 = 0.648

20 0.20 4.0 -0.8 0.64 0.64 • 0.20 = 0.128

21 0.30 6.3 0.2 0.04 0.04•0.30 = 0.012

22 0.20 4.4 1.2 1.44 1.44 • 0.20 = 0.288

23 0.10 2.3 2.2 4.84 4.84 • 0.10 = 0.484

𝜇 =∑ ∙ ( ) = ∑( − ) ∙ ( ) = 1.56
20.8
Step 5. Get the square root of the variance to get the standard deviation.
The variance of the probability distribution is 1.56.
The standard deviation is 𝜎 .

C. Number of Heads
When three coins are tossed, the probability distribution for the random variable X representing the
number of heads that occur is given below. Compute the variance and standard deviation of the
probability distribution.

Number of Probability
Heads
P(X) 𝑿 𝑿−𝜇 (𝑿 – 𝜇)² (𝑋 − 𝜇)² • 𝑷(𝑿)
X

26
1

𝜇 =∑ ∙ ( ) = _____ 𝜎² = ∑( − ) ∙ ( ) = _____________

The variance of the probability distribution is _____. The


standard deviation is _______.
D. Number of Cakes
A Bake shop has the following schedule of daily demand for cakes. Find the expected number of cake demands
per day.
Number of cake demands (in hundred) Probability P
X (X)
0 0.10
1 0.15
2 0.20
3 0.25
4 0.20
5 0.10

Number of Probability
cake
P(X) 𝑿 𝑿−𝜇 (𝑿 – 𝜇)² (𝑋 − 𝜇)² • 𝑷(𝑿)
demands

0 0.10

1 0.15

2 0.20

3 0.25

4 0.20

5 0.10

𝜇 ) = ___ 𝜎² ) = _____________

The variance of the probability distribution is _____. The


standard deviation is _______.

IV. What I Have Learned


Answer the following questions. Write your answers on the space provided.
1. How do you interpret the variance of a probability distribution?
__________________________________________________________________________________
_________________________________________________________________________________.
27
2. How do you solve problems involving variance of probability distribution? Give the step by step procedure.
__________________________________________________________________________________
_________________________________________________________________________________.

V. Reflection

1. What have you learn from this activity?


______________________________________________________________________________________
__________________________________________________________________.
2. Have you had difficulty in dealing with answering the activities?
______________________________________________________________________________________
_____________________________________________________________________._________________
____________________________________________.
3. What do you like the most with this activity sheet?
______________________________________________________________________________________
____________________________________________________________________.__________________
___________________________________________.
VI. References

Belecina, Rene R. et. al. Statistics and Probability. P. Florentino ST., Sta. Mesa Heights, Quezon City: Rex
Printing Company, Inc., 2016

Retrieved from https://amsi.org.au/ESA_Senior_Years/SeniorTopic4/4c/4c_2content_6.html

2.6 Prepared by:


𝜎² = ∑(𝑿− 𝝁)𝟐) ∙ 𝑷(𝑿 = 2.14 ) =
𝜇 = ∑𝑋 ∙ 𝑃(𝑋
AMIE-LYN S. CEDO
0.576 5.76 2.4 0.50 0.10 5
VII. Answer Key
0.392 1.96 1.4 0.80 0.20 4

28
Name: ______________________________________ Grade: _______________________
Section: _____________________________________ Date: ________________________

LEARNING ACTIVITY SHEET


ILLUSTRATING NORMAL DISTRIBUTION

II. BACKGROUND
This activity sheet serves a guide to the learners. This is an introduction about the normal distribution.
You’ve learned from the previous Learning Activity Sheet the concept of random variables that
focuses on discrete probability distribution. The probability of a distribution enables to make
predictions and decisions concerning the populations. However, there are data that are continuous.
The distribution of this type is known as the normal
probability distribution or the normal curve.
Data can be distributed in a variety of
ways. It can spread more on the left or more on the
29
right or even jumbled up. But there are many cases where the data tends to be around a central
value with no bias left or right, and it gets close to a "Normal Distribution". Lo ok a t the model
shown below.

Fig. 1. The Normal Probability Distribution


The given distribution consists of a large number of cases and the three measures of
averages (mean, median, and mode) are equal and the distribution is symmetrical and the skewness
is 0. In Statistics, such distribution is called normal distribution or simply normal curve.The normal
distribution is the most important and most widely used distribution in statistics. It is the most
common type of data distribution and is also known as bell curve. Most observations are centered
around the mean and fewer data
observation beyond either side of the
mean.

Properties of the Normal Probability


Distribution
The normal probability distribution has the
following properties
1. The distribution curve is bell-shaped.
2. The curve is symmetrical about
its center. The mean,
the median, and the mode are equal and coincide at the center.
3. The width of the curve is determined by the standard deviation of the
distribution.
4. The tails of the curve flatten out indefinitely along the horizontal axis, always
approaching the axis but never touching it. That is, the curve is asymptotic to the
base line.
5. The area under the curve is 1. Exactly half of the values are to the
left of the center and exactly half the values are to the right.
6. Empirial rule, it has a distribution with 68% of observation in the
population lie within 𝜇 ± 𝜎, 95% of observations in the population lie
within 𝜇 ± 2𝜎 and
99.7% of observations in the population lie within 𝜇 ± 3𝜎.

The Normal Random Variable


The equation that describes normal curve is:

𝑓
𝜎√2𝜋

Where:
µ = constant, , −∞ < 𝜇 < +∞ (mean of X)
30
𝜎 = constant, 𝜎 > 0 (standard deviation of X)
𝜋 = 3.1416
ℯ = 2.7183

The Standard Normal Distribution


A normal distribution is determined by the mean 𝜇 and standard deviation 𝜎. If the mean µ = 0
and a standard deviation σ = 1, the normal distribution is called a standard normal distribution. By
substituting the mean, µ = 0 and the standard deviation, σ = 1 in the formula mathematicians are
able to find the areas under the normal curve. The areas under the normal curve can be found using
the Areas under the Standard Normal Distribution Table. These areas have already been
predetermined for use.

Mean
Fig. 2.2.The Areas Under the Normal Curve
The middle regions under the normal curve are shown above. The total area between z = −1
and z =1 is 2(0.3413)=0.6826 or 68.26%; between 𝑧 = −2 and 𝑧 = 2, the total area is 0.9544 or
95.44%; and the total area between 𝑧 = −3 and 𝑧 = 3 is 0.9974 or 99.74%. This means that we can
determine the area in any specified region under the normal curve and associate it with probability,
proportion and percentage.

Using a Standard Normal Distribution Table

The table of areas under the normal curve is also known as the z-Table. The z-score is a
measure of relative standing. It is calculated by subtracting 𝑥 or 𝜇 from the measurement of 𝑥 and
then dividing the result by 𝑠 or σ. The final result, the z-score, represents the distance between a
given measurement 𝑥 and the mean, expressed in standard deviations. Either z-score locates 𝑥
within a sample within a population.

z .00 .01 .02 .03 .04 .05 .06 .07 .08 .09
0.0 0.0000 0.0040 0.0080 0.0120 0.0160 0.0199 0.0239 0.0279
0.0319 0.0359 0.1 0.0398 0.0438 0.0478 0.0517 0.0557 0.0596
0.0636 0.0675 0.0714 0.0753 0.2 0.0793 0.0832 0.0871 0.0910
0.0948 0.0987 0.1026 0.1064 0.1103 0.1141 0.3 0.1179 0.1217
0.1255 0.1293 0.1331 0.1368 0.1406 0.1443 0.1480 0.1517 0.4
0.1554 0.1591 0.1628 0.1664 0.1700 0.1736 0.1772 0.1808 0.1844
0.1879 0.5 0.1915 0.1950 0.1985 0.2019 0.2054 0.2088 0.2123
0.2157 0.2190 0.2224 0.6 0.2257 0.2291 0.2324 0.2357 0.2389
0.2422 0.2454 0.2486 0.2517 0.2549 0.7 0.2580 0.2611 0.2642
0.2673 0.2704 0.2734 0.2764 0.2794 0.2823 0.2852
0.8 0.2881 0.2910 0.2939 0.2967 0.2995 0.3023 0.3051 0.3078 0.3106 0.3133
0.9 0.3159 0.3186 0.3212 0.3238 0.3264 0.3289 0.3315 0.3340 0.3365 0.3389
1.0 0.3413 0.3438 0.3461 0.3485 0.3508 0.3531 0.3554 0.3577
0.3599 0.3621 1.1 0.3643 0.3665 0.3686 0.3708 0.3729 0.3749

31
0.3770 0.3790 0.3810 0.3830 1.2 0.3849 0.3869 0.3888 0.3907
0.3925 0.3944 0.3962 0.3980 0.3997 0.4015 1.3 0.4032 0.4049
0.4066 0.4082 0.4099 0.4115 0.4131 0.4147 0.4162 0.4177 1.4
0.4192 0.4207 0.4222 0.4236 0.4251 0.4265 0.4279 0.4292 0.4306
0.4319 1.5 0.4332 0.4345 0.4357 0.4370 0.4382 0.4394 0.4406
0.4418 0.4429 0.4441 1.6 0.4452 0.4463 0.4474 0.4484 0.4495
0.4505 0.4515 0.4525 0.4535 0.4545 1.7 0.4554 0.4564 0.4573
0.4582 0.4591 0.4599 0.4608 0.4616 0.4625 0.4633 1.8 0.4641
0.4649 0.4656 0.4664 0.4671 0.4678 0.4686 0.4693 0.4699 0.4706
1.9 0.4713 0.4719 0.4726 0.4732 0.4738 0.4744 0.4750 0.4756 0.4761 0.4767
2.0 0.4772 0.4778 0.4783 0.4788 0.4793 0.4798 0.4803 0.4808
0.4812 0.4817 2.1 0.4821 0.4826 0.4830 0.4834 0.4838 0.4842
0.4846 0.4850 0.4854 0.4857 2.2 0.4861 0.4864 0.4868 0.4871
0.4875 0.4878 0.4881 0.4884 0.4887 0.4890 2.3 0.4893 0.4896
0.4898 0.4901 0.4904 0.4906 0.4909 0.4911 0.4913 0.4916 2.4
0.4818 0.4920 0.4922 0.4925 0.4927 0.4929 0.4931 0.4932 0.4934
0.4936 2.5 0.4938 0.4940 0.4941 0.4943 0.4945 0.4946 0.4948
0.4949 0.4951 0.4952 2.6 0.4953 0.4955 0.4956 0.4957 0.4959
0.4960 0.4961 0.4962 0.4963 0.4964 2.7 0.4965 0.4966 0.4967
0.4968 0.4969 0.4970 0.4971 0.4972 0.4973 0.4974 2.8 0.4974
0.4975 0.4976 0.4977 0.4977 0.4978 0.4979 0.4979 0.4980 0.4981
2.9 0.4981 0.4982 0.4982 0.4983 0.4984 0.4984 0.4985 0.4985 0.4986 0.4986
3.0 0.4987 0.4987 0.4987 0.4988 0.4988 0.4989 0.4989 0.4989
0.4990 0.4990 3.1 0.4990 0.4991 0.4991 0.4991 0.4992 0.4992
0.4992 0.4992 0.4993 0.4993 3.2 0.4993 0.4993 0.4994 0.4994
0.4994 0.4994 0.4994 0.4995 0.4995 0.4995 3.3 0.4995 0.4995
0.4995 0.4995 0.4996 0.4996 0.4996 0.4996 0.4996 0.4997 3.4
0.4997 0.4997 0.4997 0.4997 0.4997 0.4997 0.4997 0.4997 0.4997
0.4998
3.5 0.4998 0.4998 0.4998 0.4998 0.4998 0.4998 0.4998 0.4998 0.4998 0.4998

Example 1:

Eleana conducted a survey and gathered data regarding daily travel time in minutes. After assuming
the data is normally distributed, she found out that the daily travel time has a mean of 32.28
minutes with a standard deviation of 10.68 minutes.

µ = mean daily travel time in minutes standard deviation of daily time travel in minutes
𝜎 = standard deviation of daily time travel in minutes
X = daily time travel in minutes
The data is assumed to be normally distributed, hence, it has a symmestric property. Since mean
µ = 32.28, therefore the mode = median = 32. 28

Example 2:

Teacher Jared conducted a 100-item pre-test to his 30 students. He then marks the tests and
gathered the scores of his students. Assuming that the population is normally distributed, 95% of the
students got a score between 70 point and 89 points.

A is a normal continuous random variable which is equal to the exam scores. The variable A
may take on any value within the range of (70,89). Some may have exam scores that is exactly
70 or 89 or even other integer value within that range. However, some may have scores
that have a decimal value that is still within the said range. There are infinitely many possible values
of A that’s within the set range.
32
Computing for the mean:
The mean is halfway between 70 and 89

𝜇= = 79.5
Therefore, the mean exam scores of the students are equal to 79.5
Computing for the standard deviation:

95% is 2 standard deviation (2𝜎) either side of mean, meaning a total of 4 standard deviation.

𝜎= = 4.75
Therefore, the standard deviation of exams of the students is equal to 4.75.
Example 3:

The length (in days) of a randomly chosen human pregnancy is a normal random variable with
𝜇 = 266, 𝜎= 16. So, X = length of pregnancy in days. What is the probability that a randomly chosen
pregnancy will last less than 246 days?

We want 𝑃(𝑋 < 246). To find this probability, we first convert 𝑋= 246 to a Z-

score.
𝑍= = = −1.25
This is the area under the normal probability curve to the
left of 𝑍 = −1.25

Thus, the probability that a randomly chosen pregnancy lasts less than 246 days is 0.1056. In other
words, there is an 11% chance that a randomly selected pregnancy will last less than 246 days.

33
III. LEARNING COMPETENCY
1. Define a normal random variable
2. Identify the normal probability distribution function
3. Illustrate normal random variable and its characteristics (M11/12SP-IIIc-1)

IV. ACTIVITIES
A. Many things closely follow a normal distribution. Give five examples of real-life data which follow this
type of pattern. Example: heights of people
1. _____________________________________________
2. ______________________________________________
3. ______________________________________________ 4.
______________________________________________
5. ______________________________________________

B. Encircle the correct answer.


1. Children’s height are normal with 𝜇 = 36 inches and 𝜎 = 2 inches. So a child 39.5 inches tall has a
standardized height of ____.
A. 1.75 B. 18 C. 1.10 D. none of the choices
2. In February 2021, 1,664,479 persons took the COVID vaccine in Asia. The distribution of Pfizer vaccine had
a 𝜇 = 496 and a 𝜎 = 144. Find 𝑍 if X = 366.
A. 1.14 B. 4.35 C. 2.54 D. none of the choices
3. Now that there is a pandemic, some doctors believe that a person can be healthy if he or she can lose five
pounds, on the average, in a month by reducing his or her fat intake and by exercising regularly. Suppose a
person lost ten pounds in a month. Use a 𝜎 = 2. What is 𝑍?
A. 2.5 B. -2.5 C. 5 D. none of the choice

C. Fill in the blanks with the appropriate word or phrase to make a meaningful statement.
1. The distribution curve is a ________________ probability distribution.
2. The area under the curve is ________________.
3. The width of the curve is determined by the ________________ of the distribution.
4. The curve is ________________ to the base line.
5. The curve is ________________ about its center.
6. The mean, median, and mode of the normal curve are ________________.
7.The standard normal curve is a normal probability distribution that has a mean of _______________
and standard deviation of _________________.

V. GUIDE QUESTIONS:

1. What did you understand about normal distribution?


___________________________________________________________________________
2. What are the characteristics of a normal distribution?
___________________________________________________________________________________
______________________________________________________.

VI. REFLECTION:

1. What have you learn from this activity?


___________________________________________________________________________________
_________________________________________________________________.

34
2. Have you had difficulty in dealing with answering the activities?
___________________________________________________________________________________
________________________________________________________________.

3. What do you like the most with this activity sheet?


___________________________________________________________________________________
__________________________________________________________________.

VII. REFERENCES:

Belecina, Rene, E. Baccay, and E. Mateo. Statistics and Probability. Manila: Rex Book Store, Inc., 2016.

Glen, Stephanie. "Normal Distributions (Bell Curve): Definition, Word Problems".


StatisticsHowTo.com: Elementary Statistics for the rest of us! Accessed May 22, 2020.
https://www.statisticshowto.com/probability-and-statistics/normal distributions/

https://prezi.com/r3kegqwojjpd/illustrating -a-normal-random-variable-and-its-characteristics/

https://prezi.com/xsr8u2zrs3dl/normal -distribution/

https://courses.lumenlearning.com/wmopen-concepts-statistics/chapter/introduction-to-normalrandom-
vairables-6-of-6/

https://courses.lumenlearning.com/introstats1/chapter/the-standard-normal-distribution/

Prepared by:
AMIE-LYN S. CEDO

VIII. Answer Key:

7. 0 , 1
6. Equal
. Symmetric 5
. Asymptotic 4
. Standard deviation 3
. 1 2
. Bell-shaped C.1
. A 3
. A 2
.A B.1
Learner’s answer may vary A.

35
Name of Learner: ____________________________Grade Level: ______________________
Section: ____________________________________ Date: ____________________________

LEARNING ACTIVITY SHEET


IDENTIFYING AREAS UNDER THE NORMAL CURVE

.
In the previous lesson, you have learned about the properties of the normal distribution. The
distribution curve is bell-shaped, symmetrical about its center. The mean, the median, and the
mode are equal and coincide at the center, the width of the curve is determined by the standard
deviation of the distribution, the curve is asymptotic to the base line, the area under the curve s 1
and it represents the probability or proportion or the percentage associated with the specific sets
of measurement values.

You have also learned how to read entries in the z-table and how to use it in determining the area
under the normal curve. This time, you will enrich your learning on identifying regions under the
normal curve corresponding to different standard normal values. When you say region under the
curve, you are interested in the area of the region.

I. Background Information for Learners

The Standard Normal Distribution normally distributed variable has its own
mean and standard deviation, as stated earlier, the shape and location of these
curves will vary. In practical applications, then, you would have to have a table of
areas under the curve for each variable. To simplify this situation, statisticians use
what is called
the standard normal distribution.

The curve shows a shaded region between 𝑧 = 0and 𝑧 = 1.


Similarly, a region between 𝑧 = 0 and 𝑧 = −1 has the same area.

The shaded region between z= 0 and z= 2 is the same with the


region between z=0 and z=-2

Regions under the curve can be described in terms of area. Area between two specific
zvalues can be determined using the z-table and the suggested steps below.

Steps in Identifying Region under a Normal Curve:

36
1.Draw the normal curve and locate the given z-value or values at the base line of the curve. Then, draw
a vertical line through the given z-value or values and shade the required region.
2. Use the z-table to find the areas that correspond to the given z-value or values.
3. Perform appropriate operations to get the required area, if needed.
4. Write the required area.

Case 1. When the required area is between 𝑧 = 0 and any z-value:

Example 1: Find the area that corresponds to 𝑧 = −1 is the same as finding the area between 𝑧 = 0and 𝑧
= −1.
1. Draw the normal curve and locate the given
zvalue or values at the base line of the curve.
Then, draw a vertical line through the given
zvalue or values and shade the required region.

2. Use the z-table to find the areas that 𝑧 = −1 corresponds to an area of 0.3413
correspond to the given z-value or values.

3. Perform appropriate operations to get the No operations needed because the answer
required area, if needed. is already given in the z-table.

4. Write the required area. Thus, the area that corresponds to 𝑧 = −1


is 0.3413.

Example 2: Find the area that corresponds to 𝑧 = 1.

1. Draw the normal curve and locate the given z- value or


values at the base line of the curve. Then, draw a vertical line
through the given z value or values and shade the required
region.
2. Use the z-table to find the areas that 𝑧 = 1 corresponds to an area of 0.3413
correspond to the given z-value or values.
3. Perform appropriate operations to get the No operations needed because the answer
required area, if needed. is already given in the z-table
4. Write the required area. Thus, the area that corresponds to 𝑧 = 1 is
0.3413.

Remember: When z is negative, simply ignore the sign. The negative informs us that the region is
found on the left side of the mean. Remember, areas are positive values.

Case 2. When the required area is greater than z

37
Other than the terms “greater than”, you can use these words to convey the same meaning: at least z,
more than z, to the right of z, or above z.
Example 3: Find the area above𝑧 = −1.34.

1. Draw the normal curve and locate the given z- value


or values at the base line of the curve. Then, draw a
vertical line through the given z value or values and
shade the required region.

2. Use the z-table to find the areas that 𝑧 = −1.34 corresponds to an area of 0.4099
correspond to the given z-value or values.
3. Perform appropriate operations to get the The graph suggests addition. 0.4099 + 0.5 =
required area, if needed. 4. Write the required 0.9099
area. Thus, the area above 𝑧 = −1.34 is 0.9099.

Example 4: Find the area to the right of 𝑧 = 1.56

1. Draw the normal curve and locate the given z- value


or values at the base line of the curve. Then, draw a
vertical line through the given z value or values and
shade the required region.
2. Use the z-table to find the areas that 1.56 corresponds to an area of 0.4406.
correspond to the given z-value or values.
3. Perform appropriate operations to get theThe graph suggests subtraction. 0.5 − 0.4406
required area, if needed. = 0.0594
4. Write the required area. Thus, the area to the right of 𝑧 = 1.56 is
0.4406.

Case 3. When the required area is less than z

Other than the terms “less than”, you can use these words to convey the same meaning: at most z, no
more than z, not greater than z, or to the left of z

Example 5: Find the area to the left of 𝑧 = −1.52

38
1. Draw the normal curve and locate the given z- value
or values at the base line of the curve. Then, draw a
vertical line through the given zvalue or values and
shade the required region.
2. Use the z-table to find the areas that 𝑧 = −1.52 corresponds to an area of 0.4357.
correspond to the given z-value or values.
3. Perform appropriate operations to get theThe graph suggests subtraction. 0.5 − 0.4357
required area, if needed. = 0.0643
4. Write the required area. Thus, the area to the left of 𝑧 = −1.52 is
0.0643.

Example 6: Find the area less than 𝑧 = 1.25

1.Draw the normal curve and locate the given z- value or


values at the base line of the curve. Then, draw a vertical
line through the given zvalue or values and shade the
required region.

2. Use the z-table to find the areas that 𝑧 = 1.25 corresponds to an area of 0.3944
correspond to the given z-value or values.
3. Perform appropriate operations to get the The graph suggests addition. 0.5 + 0.3944 =
required area, if needed. 0.8944
4. Write the required area. Thus, the area less than 𝑧 = 1.25 is 0.8944.

Case 4. When the required area is between two z-values of the same sign.

Example 7: Find the area between 𝑧 = 1.70 and 𝑧 = 0.82.

1. Draw the normal curve and locate the given z- value or


values at the base line of the curve. Then, draw a vertical
line through the given z value or values and shade the
required region.
2. Use the z-table to find the areas that 𝑧 = 1.70 corresponds to an area of 0.4554
correspond to the given z-value or values. 𝑧 = 0.82 corresponds to an area of 0.2939
3. Perform appropriate operations to get the The graph suggests subtraction. Subtract the
required area, if needed. smaller area from the larger area. 0.4554 −
0.2939 = 0.1615
4. Write the required area. Thus, the area between 𝑧 = 0.82 and 𝑧 = 1.70
is 0.1615.

39
Example 8: Find the area between 𝑧 = −2 and 𝑧 = −1

1. Draw the normal curve and locate the given z- value


or values at the base line of the curve. Then, draw a
vertical line through the given zvalue or values and
shade the required region.
2. Use the z-table to find the areas that 𝑧 = −2 corresponds to an area of 0.4772 𝑧 =
correspond to the given z-value or values −1 corresponds to an area of 0.3413
3. Perform appropriate operations to get the The graph suggests subtraction. Subtract the
required area, if needed. smaller area from the larger area. 0.4772 −
0.3413 = 0.1359
4. Write the required area. Thus, the area between 𝑧 = −2 and 𝑧 = −1 is
0.1359.

Case 5. When the required area is between −z1 and z2

Example 9: Find the area between 𝑧 = −1.54 and z = 1.75

1.Draw the normal curve and locate the given z- value or


values at the base line of the curve. Then, draw a vertical
line through the given z value or values and shade the
required region.

2. Use the z-table to find the areas that 𝑧 = −1.54 corresponds to an area of 0.4382 𝑧
correspond to the given z-value or values. = 1.75 corresponds to an area of 0.4599
3. Perform appropriate operations to get the The graph suggests addition. 0.4382 + 0.4599
required area, if needed. = 0.8981
4. Write the required area Thus, the area between 𝑧 = −1.54 and 𝑧 =
1.75
is 0.8981.

Let’s extend your learning!


Now, let us extend your learning on representing the region under the normal
curve in terms of percent. Sketch the normal curve and use the z-table to find the
area that corresponds to each of the following z-values. Then, multiply the area to
100 to get
its percent form. The final answers are given as your guide.

1.Find the area between z= 1.68 and z =-1.37.

40
1. Draw the normal curve and locate the given z- value
or values at the base line of the curve. Then, draw a
vertical line through the given zvalue or values and
shade the required region.
2. Use the z-table to find the areas that z= 1.68 corresponds to an area of 0.4535
correspond to the given z-value or values. z= -1.37 corresponds to an area of 0.1443
3. Perform appropriate operations to get the The graph suggests Addition.
required area, if needed. To convert area to 0.4535+ 0.1443= 0.8682
percent, multiply the area to 100.
4. Write the required area (in percent). Thus, the area between 𝑧 = 1.68 and 𝑧 = -
1.37 is 86.82%

2. Find the area to the left of z= 2.06.

1. Draw the normal curve and locate the given z-


value or values at the base line of the curve.
Then, draw a vertical line through the given zvalue
or values and shade the required region.

2. Use the z-table to find the areas that z= 2.06 corresponds an area of 0.4803
correspond to the given z-value or values.
3. Perform appropriate operations to get the The graph suggest addition.
required area, if needed. To convert area to 0.5 + 0.4803= 0.9803
percent, multiply the area to 100.
4. Write the required area (in percent). Thus, the area to the left of z=2.06 is
98.03%

Let’s summarize!

• When speaking about a region under the curve, we are interested in the area of the
region. • The area in any specified region under the normal curve can be associated with
probability, proportion, or percentage.
• When 𝑧 is negative, simply ignore the negative sign and proceed. The negative sign
indicates that the region is on the left side of the mean or below the mean. Areas are always
positive value

Learning Competency:
1. Identifies region under the normal curve corresponding to different standard normal vales.
M11/12SP-111C3
2. Find and interpret the area under a normal curve.

41
ACTIVITY 1
WHAT I CAN DO

Direction: Answer the question below.


1. What is the total area under the standard normal distribution curve?
_________________________________________________________________
2. What percentage of the area falls below the mean? Above the mean?
_________________________________________________________________
_______________________________________________________________________

3. About what percentage of the area under the normal distribution curve falls within 1
standard deviation above the mean? 2 standard deviations below the mean? 3 standard
deviations above the mean?
________________________________________________________________________

ACTIVITY 2 FIND ME!


Direction: Answer the following questions based on the data.

2000 Grade 11 students of MORMS took a Statistics test. The scores were distributed normally
with a mean of 70 and Standard deviation of 5. Label the mean and three standard deviation from the
mean.

1.What percentage of scores are between 65 and 75?


2.What percentage of scores are between 60 and 70?

42
3.What percentage of scores less than a score of 60?
4.What percentage of scores greater than a score of 80?
5. What percentage of score are between 70 and 75?

ACTIVITY 3
DRAW ME
Direction: Draw the normal curve, then locate the given z values in the curve. Find the area that
corresponds to the given z values using the z-table, then shade the required area.

1. Between z= 0 and z= 1.77


2. To the right of z = 2.01
3. To the left of z=- 0.75
4. Between z= 0 and z= 0.75
5. To the left of z =1.39

Reflection

1. What have you learned from this activity?


_____________________________________________________________________
2. What did you like most from this lesson?
_____________________________________________________________________
3. What is the most challenging or least interesting from the activities?
____________________________________________________________________________

References for learners:

Belecina, Rene, E. Baccay, and E. Mateo (2016). Statistics and Probability: Rex Book Store, Inc., Manila
Philippines.
Glen, Stephanie "Normal Distributions (Bell Curve): Definition, Word Problems".
StatisticsHowTo.com: Elementary Statistics for the rest of us!Retrived from URL on May 22,.
Bluman, Allan G. (2012) Elementary Statistics:A Step by Step Approach, Eighth Edition, McGraw-Hill Companies
Inc., 1221 Avenue of the Americas, New York, pp.345-347

43
44
Date: ______________________ Section: ______________________________________
Grade Level: _________________ Name of Learner: _____________________________
T-III, MORMS
CHARMIE G. RELLORES
Prepared by:
Answer
Key:
Activity
1 1 The total area under the standard normal curve is 1.00
. 100or %
50 2 % above the mean, 5o % below the mean.
1 3. standard deviation above the mean has an area of
. 34.13 %.
2 standard deviation below the mean
area has
of an
47.72 %.
3 standard deviation above the mean has an area of
50.00%
Activity
2
55 60 65 70 75 85
80
1 Between 65 &68.26
75? %
.2 Between 60 &47.72
70? %
3. Less than 60? 2.28 %
4. Greater than 80?
2.28 %
5. Between 70 and
34.13
75?%
.
Activity
3 1 Between z=0 and z=1.77
46.16 %
2. To the right of z= 2.01
2.22 %
3. To the left
0.75
of- z= 22.66 %
.4 Between z= 0 and 27.34
z=0.75%
5. To the left of z=1.39
91.77 %
.
LEARNING ACTIVITY SHEET
CONVERTING NORMAL RANDOM VARIABLE TO A STANDARD NORMAL VARIABLE
AND VICE VERSA

In the previous lessons, you have learned about the basic concept of normal distribution. You
further learned on how to identify the regions under the normal curve using the z-table. In this
lesson, you will learn how to relate the concept of the normal curve to a random variable distribution
using the z-score. You will be transforming a normal random variable to a standard normal variable
and vice-versa.

II. Background Information for Learners

What do you mean by standard normal distribution?


A standard normal random variable Z has a normal distribution with mean =0 and standard deviation =1.

Raw scores may be composed of large values, but large values cannot be accommodated at the
base line of the normal curve. So, they must be transformed into scores for convenience without
sacrificing meanings associated with the raw scores. If you wish to find the proportion of area,
percentage or probability associated with a raw score, you must find its matched z-value using the
zscore formula. Then, the z-value leads to the area under the normal curve found in the z-table, which
is a probability or the desired percentage.
Let’s explore the concept of z-scores.

Z-score or standard score measures how many standard deviation a given value (x) is above
or below the mean. Z-scores are useful in comparing observed values. If a z-score is equal to 0, it is on
the mean. A positive z-score indicates that the score or observed value is above the mean, whereas a
negative z-score indicates that the score or observed value is below the mean.

For example, if a z-score is equal to 1, it is 1 standard deviation above the mean. If a z-score is equal
to −2, it is 2 standard deviations below the mean.

45
To solve the application problems in this section, transform the values of the variable to z
values and then find the areas under the standard normal distribution. The areas under the normal
curve are given in terms of z-scores. Either it locates x within a sample or within a population.

The formula for calculating z is:

𝑋−𝜇
For Sample:𝑧 =
𝜎

𝑋−𝑋̅
For Population: 𝑧 =
𝑠
where:z = standard score
Χ= raw score
𝑋̅ = sample mean s = sample
standard deviation
𝜇= population mean
𝜎 = population standard deviation

Study the following examples.

Example 1: Find the z-value that corresponds to Mathematics test score of 55 given the mean,
=50and the standard deviation, =2.

Thus, the z-value that corresponds to the raw score 55 is 2.5 in a population distribution.

1. Choose the formula to use. (Use the


zscore formula for population data as 𝑋−𝜇
indicated in the problem 𝑧= 𝜎

= 55; = 50; =2
2. Write the given values.
𝑋−𝜇
𝑧=
𝜎
55 − 50 5
= =
3. Substitute the given values in the 2 2
computing formula. Then, compute the
z-value. 𝑧 = 2.5
This means the score 55 is 2.5 standard deviations above the mean.

Example 2: On the midterm examination in Statistics, the sample mean was 80 and the sample
standard deviation was 7. Determine the standard score of a student who got a score of 75 assuming
that the scores are normally distributed.

46
𝑋 − 𝑋̅
1. Choose the formula to use. (Use
𝑧=
the z-score formula for sample 𝑠
data as indicated in the problem.)

2. Write the given values. Χ = 75; 𝑋̅= 80; s= 7

3. Substitute the given values in the 𝑋 − 𝑋̅


computing formula. Then, 𝑧=
compute the z-value. 𝑠
75 − 80 −5
= =
7 7

𝑧 = −0.71

Thus, the z-value that corresponds to the raw score 75 is −0.71 in a sample distribution. This
means the score 75 is 0.71 standard deviations below the mean.

Example 3. Cassey scored 78 on her History test which had a mean of 70 and a standard deviation of 3
and she scored 84 on her Math test which had a mean of 80 and a standard deviation of 2, on which
29 test did she score better? In which subject was her standing better, assuming that the scores
in her Mathematics and History class are normally distributed?

1. Choose the formula to use. (For 𝑋−𝜇


population data) 𝑧=
𝜎
Subject Cassey mean Standard
Score (𝜇) deviation(𝜎)
2. Write the given values. History 78 70 3
Math 84 80 2

3. Substitute the given values in the History


computing formula. Then, Mathematics
compute the z-value. 𝑋−𝜇 s 𝑋−𝜇
𝑧= 𝑧=
𝜎 𝜎

z= z=
8
z=
z=
2
3
𝑧=2
𝑧 = 2.67
The z-value that corresponds to Cassey’s History raw score of 78 is 2.67 in a population
distribution. This means the score 78 is 2.67 standard deviation above the mean. Meanwhile, the z-
value that corresponds to her Mathematics score of 84 is 2. This means that 84 is 2 standard
deviations above the mean. Thus, Cassey has a better standing in History compared to Mathematics
47
Let’s extend your learning!

You already know how to convert a random normal variable to a standard


normal score or z-score. This time let’s do the reverse. Given the z-score,
compute for the raw scores.

1. Given: 𝜇 = 30, 𝜎 = 5. What is the raw score when 𝑧 = 1.30?


1. Use the computing formula for finding the
zscore for population data. You can derive 𝑥−𝜇
the formula for easy computation. 𝑧=
𝜎
𝑥−𝜇
𝜎(𝑧) = ( )𝜎
𝜎

𝑧𝜎 = 𝑥 − 𝜇
𝑥 = 𝜇 + 𝑧𝜎
2. Write the given values.
𝜇 = 30;𝜎 = 5, 𝑧 = 1.30
3 . Substitute the given values in the 𝑥 = 𝜇 + 𝑧𝜎
computing formula. Then, = 30 + 1.30(5)
compute the raw score ( ). = 30 + 6.5
𝑥 = 36.5

Thus, the raw score when 𝐳 = 𝟏. 𝟑𝟎 is


36.5.

2 . Given: 𝜇 = 64=, 𝜎 = 7. What is the raw score when 𝑧 = 0.76?

1.Use the computing formula for finding the z- 𝑥−𝜇


score for sample data. You can derive the formula 𝑧=
for easy computation. 𝜎

𝑥−𝜇
𝜎(𝑧) = ( )𝜎
𝜎

𝑧𝜎 = 𝑥 − 𝜇
𝑥 = 𝜇 + 𝑧𝜎

2. Write the given values. 𝜇 = 64;𝜎 = 7, 𝑧 =0.76

48
3. Substitute the given values in the computing 𝑥 = 𝜇 + 𝑧𝜎
formula. Then, compute the raw score ( ). = 64 + 0.76(7)
= 64 + 5.32
𝑥 = 69.32

Thus, the raw score when 𝒛 = 𝟎. 𝟕𝟔 is 69.32

Let’s Summarize!

The formula for calculating z is:


𝑋−𝜇
For Sample:𝑧 =
𝜎 𝑋−𝑋̅
For Population: 𝑧 =
𝑠

• Z-score or standard score measures how many standard deviation a given value (x) is above or below
the mean.
• If a z-score is equal to 0, it is on the mean. A positive z-score indicates that the score or observed value
is above the mean, whereas a negative z-score indicates that the score or observed value is below the
mean.

Learning Competency:
1. Convert a normal random variable to a standard normal variable and vice versa (MII/12SP-

IIIc4)
2. Solve word problems involving z-scores.
3.

𝑿 𝝁 𝝈 𝒛 ACTIVITY 1
1 23 32 8 ? COMPLETE ME
Direction:
2 250 231 120 ?
Complete
3 98 ? 5 2.21 the table
4 ? 450 15 -1.5 below.
Show
your solution.

49
Activity 2
WHAT I CAN DO
Direction: Calculate the z-score, given the mean, standard deviation, and raw score (X).

1. Scores on a history test have an average of 80 with a standard deviation of 6. What


is the z-score for a student who earned a 75 on the test?
2. The weight of chocolate bars from a particular chocolate factory has a mean of 8 ounces with a
standard deviation of .1 ounce. What is the z-score corresponding to a weight of 8.17 ounces?
3. Books in the library are found to have an average length of 350 pages with a standard deviation
of 100 pages. What is the z-score corresponding to a book of length 80 pages?
4. The temperature is recorded at 60 airports in a region. The average temperature is 67 degrees
Fahrenheit with a standard deviation of 5 degrees. What is the z-score for a temperature of 68
degrees?

Activity 3
SOLVE ME
Direction: To solve the application problems in this section, transform the values of the variable to z
values.

1. Alex scored 90 during the first periodic exam in Mathematics and 88 during the second periodic exam. The
scores in first periodic exam have a mean 𝜇 = 83 and a standard deviation 𝜎 = 9. Scores in the second
periodic exam have a mean 𝜇 = 80 and a standard deviation 𝜎 = 8. In which periodic exam was his standing
better, assuming that the scores in his periodic exams are normally distributed?
2. On a final examination in Biology, the mean was 75 and the standard deviation was 12. Determine the
standard score of a student who received a score of 60 assuming that the scores are normally distributed.

Reflection

4. What have you learned from this activity?


_____________________________________________________________________
5. What did you like most from this lesson?
_____________________________________________________________________
6. What is the most challenging or least interesting from the activities?
____________________________________________________________________________

References for learners:

Belecina, Rene, E. Baccay, and E. Mateo (2016). Statistics and Probability: Rex Book Store, Inc., Manila
Philippines.
Glen, Stephanie "Normal Distributions (Bell Curve): Definition, Word Problems".
50
51
CHARMIE G. RELLORES
Prepared by:
Activity 1
COMPLETE ME
𝑿 𝝁 𝝈 𝒛
1 23 32 8 -1.125
2 250 231 120 0.1583
3 98 86.95 5 2.21
4 427.5 450 15 -1.5
Activity 2
1. z-score =-0.83
2. z-score =1.7
3. z-score =-2.7
4. z-score =0.2
Activity 3
First periodic exam: = Secondperiodic
exam: = 1
Alex’s standing the second periodic exam better
compared to the first periodicexam.
2. Standard score of students who received score of 60
in final examination in Biology is -1.25
Key to Correction
(thoughtco.com)
Companies Inc., 1221 Avenue of the Americas, New York, pp.344-345 Practice Problems for Z-Scores
Bluman, Allan G. (2012) Elementary Statistics:A Step by Step Approach, Eighth Edition, McGraw-Hill
StatisticsHowTo.com: Elementary Statistics for the rest of us!Retrived from URL on May 22,.
T-III
(MORMS)
Name of
Learner:
___________
___________
______Grade
Level:
___________
___________
Section: ____________________________________ Date: ____________________________

LEARNING ACTIVITY SHEET


COMPUTING PROBABILITIES AND PERCENTILES UNDER THE NORMAL CURVE

Background Information for Learners

You have learned about finding the areas under the normal curve. Further,
you also learned how to compute the z-value corresponding to a raw score. You
have also seen that all areas under the normal curve can be shown as probabilities
associated with standard normal variables. In this lesson, you will practice more
your skills in determining areas under the normal curve since computing
probabilities is the same with finding the areas under the normal curve.

Finding the area of a region under the normal curve is the same as finding the probability
associated with that region. Thus, the area under the normal curve denotes probability. Hence, we
could the probability between two z-values by simply calculating the required area.

Probability Notations

Let a and b be z-score values:

• 𝑃(𝑎 < 𝑧 < 𝑏)denotes the probability that the z-score is between a and b. It is read as
“the probability that the z-score falls between 𝑧 = 𝑎 and 𝑧 = 𝑏.”

• 𝑃(𝑧 > 𝑎)denotes the probability that the z-score is greater than a. It is read as “the
probability that the z-score is greater than a.”

• 𝑃(𝑧 < 𝑎)denotes the probability that the z-score is less than a. It is read as “the
probability that the z-score is less than a.”

Recall the each case in identifying regions under the curve and the steps on determining the area
under the normal curve.

Example 1: Find the probability that the z-score is at most 𝑧 = 0.91

1. Draw the normal curve and locate the given z- value or


values at the base line of the curve. Then, draw a vertical
line through the given z value or values and shade the
required region.
2. Use the z-table to find the areas that𝑧 = 0.91 corresponds to an area of 0.3186
52
correspond to the given z-value or values.

3. Perform appropriate operations to get the The graph suggests subtraction. 0.3186 + 0.5 =
required area, if needed. 0.8186 That is, 𝑷(𝒛 < 0.91) = 𝟎. 𝟖𝟏𝟖𝟔.
4. Write the required area (or probability). Thus, the probability that the z-score is at most𝑧
− 1.78 is 0.8186.

Example 2: Find the probability that the z-score is less than 𝑧 = −1.78.

1. Draw the normal curve and locate the given z-


value or values at the base line of the curve.

Then, draw a vertical line through the given zvalue or


values and shade the required region.

2. Use the z-table to find the areas that 𝑧 = −1.78 corresponds to an area of 0.4625
correspond to the given z-value or values.

3. Perform appropriate operations to get the The graph suggests subtraction. 0.5 − 0.4625 =
required area, if needed. 0.0375 That is, 𝑷(𝒛 < −1.78) = 𝟎. 𝟎𝟑𝟕𝟓.
4. Write the required area (or probability). Thus, the probability that the z-score is less
than𝑧 − 1.78 is 0.0375.

What is a percentile?

Often the units for raw test scores are not informative. You might know
that you scored 90 out of 100 on a test but it doesn’t give you much information
of what your score means. Scores are more meaningful if percentile rank is given.
What do you mean by a percentile? If your teacher told you that your score in
test is in the 90th percentile. What does it mean?

Percentile is the score at which a specified percentage of scores in a distribution fall below.
Percentile is a measure of relative standing. It is the percent of cases that are at or below a score. It
tells you how a value compares to other values.
➢ If your teacher tells you that you scored 90th percentile, it means that 90% of the grades were lower than
yours and 10% were higher.
➢ To say a score 53 is in the 75th percentile is to say that 75% of all scores are less than 53.

In the previous lesson, you learned on how to find the area or probability under the normal curve
given a z-value. Suppose, you know the area or probability this time, how would you find the
corresponding z-score?

When you are given with the area or probability and you want to know the corresponding z-
score, locate the area at the body of the table. If the exact area is not available, take the nearest area.
Then, look up the corresponding z-value

Example 1: Find the 90th percentile of a normal curve. Finding 90 th percentile means locating an area below
the point. To begin, find the z-value located at this point.

53
Solution:
Step 1: Express the 90th percentile to decimal so that you can easily find it in the z
table, that is, 0.9000. We know that 0.9000 occupies more than half of the curve. This
occupies all the area below the mean which is 0.5 or 0.5000. This left us with an area of
0.4000 above the mean (0.9000-0.5000=0.4000).
Step 2: Locate the z-value corresponding to the area 0.4000. The z-value
corresponding to the area 0.4000 is not available, so take the nearest area. The nearest area is
0.3997 which correspond to z = 1.28. So, the 90th percentile is z=1.28

Step 3: To illustrate, draw the normal curve. Draw a line through 𝑧 = 1.28 and shade the
region below it. The shaded region is 90% of the distribution.

Example 2: Find the 95th percentile of a normal curve. Finding 95thpercentile means locating an area below
the point. To begin, find the z-value located at this point.

Solution:
Step 1: Express the 95th percentile to decimal so that you can easily find it in the z
table, that is, 0.9500. We know that 0.9500 occupies more than half of the curve. This occupies
all the area below the mean which is 0.5 or 0.5000. This left us with an area of 0.4500 above
the mean (0.9500-0.5000=0.4500).

Step 2: Locate the z-value corresponding to the area 0.4500. The z-value
corresponding to the area 0.4500 is not available, so take the nearest area. The nearest areas
are 0.4495 and 0.4505. Find the z-value corresponding to 0.4495 and 0.4505. These are z=1.64
and z=1.65. We get the average of the two z-values: 𝑧 = 1.64+1.65 2 = 𝟏. 𝟔𝟒𝟓.Thus, the 95th
percentile is z=1.645.

Step 3: To illustrate, draw the normal curve. Draw a line through 𝑧 = 1.645 and shade the
region below it. The shaded region is 95% of the distribution.

Example 3: Find the upper 2% of the normal curve. Finding the upper 2% of the normal curve means locating
an area above the point. To begin, find the z-value located at this point.

Solution:
Step1: Express the given percent to decimal so that you can easily find it in the z table,
that is, 0.0200. The upper 5% or 0.0500 means to the right of a z-value above the mean. Find
the remaining area, using the upper side of the mean (0.5000- 0.0200=0.4800).

54
Step 2: Locate the z-value corresponding to the area 0.4800. The z-value
corresponding to the area 0.4800 is not available, so take the nearest area. The nearest area is
0.4798 which corresponds to 𝑧 = 2.05. Thus, the upper 2% is above 𝒛 = 𝟐. 𝟎𝟓.

Step 3: To illustrate, draw the appropriate normal curve. Draw a line through 𝑧 = 2.05 and
shade the region above it. The shaded region is 2% of the distribution.

Let’s Summarize!

Finding the area of a region is the same as finding the probability associated with that region.

• The following are probability notations used to denote probabilities under the normal curve.

1.
𝑃(𝑎 < 𝑧 < 𝑏)denotes the probability that the z-score is between a and b.
2.
𝑃(𝑧 > 𝑎)denotes the probability that the z-score is greater than a.

3.
𝑃(𝑧 < 𝑎)denotes the probability that the z-score is less than a.

• Percentile is a measure of relative standing. It is the percent of cases that are at or below a
score. It tells you how a value compares to other values.

• When you are given with the area or probability and you want to know the corresponding z
score, locate the area at the body of the table. If the exact area is not available, take the nearest
area. Then, look up the corresponding z-value.

Learning Competency: Compute probabilities and percentiles using standard normal table (MII/12SP-
IIIc-d-1)

ACTIVITY 1
WHAT I CAN DO
Find the probabilities for each, using the standard normal distribution.

1. 𝑃(0 < 𝑧 < 1.96)


55
2. 𝑃(𝑧 > −1.43)
3. 𝑃(1.12 < 𝑧 < 1.43)
4. 𝑃(−1.23 < 𝑧 < 0)

ACTIVITY 2
FIND ME

Direction: Find each of the following percentile points and draw the appropriate normal curve. Complete
your procedures.

1. Find the 99th percentile of the normal curve.


2. Find the upper 5% of the normal curve.
3. Find the 92nd percentile rank of the normal curve.

Reflection

7. What have you learnt from this activity?


_____________________________________________________________________ 8.
What did you like most from this lesson?
_____________________________________________________________________
9. What is the most challenging or least interesting from the activities?
____________________________________________________________________________

References for learners:

Belecina, Rene, E. Baccay, and E. Mateo (2016). Statistics and Probability: Rex Book Store, Inc., Manila
Philippines.
Laerd Statistics. “How to do Normal Distributions Calculations”. Accessed May 25, 2020.
https://statistics.laerd.com/statistical-guides/normal-distributioncalculations.
Bluman, Allan G. (2012) Elementary Statistics:A Step by Step Approach, Eighth Edition, McGraw-Hill Companies
Inc., 1221 Avenue of the Americas, New York, pp.347-353

Answer Key:

56
rank is 1.41
. The92dn percentile z= 3
is above �
= � 1.645.
. The upper 5% of the normal curve 2
percentile is �
. The 99th = � 2.33. 1
ty
Activi2

. � 3 � 7
4 �(− 1.2
. � 2 � 3 <�<0)−0 .390
0
3
. � .1 <�<1.4 )−0.055
�(1
� 3 6
.2 � >−1.4 .923
�(� � )−06 0
1 �(0<�<1.9 )−0.475
1
Activity

Prepared by:

CHARMIE G. RELLORES
T-III (MORMS)

STATISTICS AND PROBABILITY

Name: __________________________________ Grade Level: _________________________


Section: _________________________________ Date: _______________________________

LEARNING ACTIVITY SHEET


ILLUSTRATING RANDOM SAMPLING
Background Information for Learners
This activity sheet in Statistics and probability is designed for you Grade 11 learners. The activities that you will
perform are organized and made simple for you to exercise independent learning.
In this particular Learning Activity Sheet (LAS), you will be able to illustrate random sampling.
Before answering the series of activities, take time to read the concepts that you need to know.

57
Random Sampling

In our everyday life, we are oriented with basic concepts in statistics. When you want to know the taste of
food your mother is cooking, you only taste only a spoonful of it. On a wider scope, researchers aim to study,
describe and infer patterns of behavior, properties and characteristics about a population. It is impractical and
inconvenient to study the whole population that is why samples are only selected as representatives.

A population is a group where members have something in common, that is, the total set of observations

that can be made. Some examples include the population of women aged 20 and above, the population of

registered voters in the province of Albay, and the population of daily maximum temperatures for the month

of April in Baguio City. A sample is a smaller group or subset of the population that you want to examine.

Sampling is a process used in statistical analysis in which a predetermined number of observations are taken

from a larger population. When each element of the population has an equal chance of being selected, it is

called random sampling.

There are different sampling methods that allow all the units in the population to have an equal chance of

being selected.

1. Simple Random/Lottery Sampling


– a sampling technique by which every member of the
population has equal
an chance to be chosen as sample. It can be done through lottery method,
table of random numbers or through the use of computers.

58
2. Systematic Random Sampling
– a sampling technique by which every member of the
population is selected with a random start.

For example, in 240 grade 11 students, you can80


only
students
chooseas members of
your sample.
To select which students will be part of the sample,
videwe
the
just
population
di
size 240 by the sample
80size
, which results .toThat
3 means every
3rdstudent among the 240
grade 11 students will be included in the sample. The researcher may also opt to start on any
randomly selected number from 1 to 240 then select
3rd element
every successively until the
desired sample size is completed.

59
3. Stratified Random Sampling
– a sampling technique that is used when the population can be
classified into groups or strata based on some characteristics.
Common criterions used for
stratification are gender, age, ethnicity, and
oeconomic
soci status.

4. Clustered Sampling
– a sampling method where the population is first divided into separate
groups called
clusters
. Then a simple random sample of clusters from the available clusters in the
populationselected.
is This sampling technique is usually used when the elements of a population
are spread over a wide geographical area.

For example, if the population is composed of all citizens residing in the Philippines, the
clusters could be citizens from
Luzon, Visayas, and Mindanao. Data is then gathered from a
selected cluster
.

Learning Competency

The learner illustrates random sampling. (M11/12SP-IIIc-2) ACTIVITY 1: RANDOM SAMPLE OR NOT?

Determine whether the process of getting a sample in each situation is by random sampling or not.

60
1. To determine the common T-shirt size his students have, Mr. Santos draw his sample from a bowl
containing the names of his students with their T-shirt sizes.
Answer: ______________

2. To determine the most liked subject in their school, Sarah interviewed the president of each class.
Answer: ______________

3. To know the most preferred learning modality of his classmates, Joseph interviewed their class
officers.
Answer: ______________

4. To determine the performance of of the SHS students in Statistics, the teacher draws 5 students from
every SHS class to take the Statistics test.
Answer: ______________

ACTIVITY 2: IDENTIFY ME!


Identify the type of random sampling illustrated by the following situations.

1. A barangay chairman wants to know whether his constituents are in favor or not on putting satellite
markets in their barangay. He wants to select a sample of 200 from his constituents, from the youths, adults
and from the senior citizens.
Answer: ______________

2. A researcher selected the participants of his study by selecting every 5th member of the population.
Answer: ______________

3. A researcher used his computer in randomly selecting a sample of n = 240 from a population of 800.
Answer: ______________

4. A researcher surveyed all teachers in each of the 12 randomly selected secondary schools in Albay.
Answer: ______________

ACTIVITY 3: THINK ABOUT IT!


Give one research situation where each of the random sampling methods is being applied.

61
1. Simple Random Sampling
___________________________________________________________________________________
___________________________________________________________________________________
________________________________________________________________________________

2. Systematic Random Sampling


___________________________________________________________________________________
___________________________________________________________________________________
________________________________________________________________________________

3. Stratified Random Sampling


___________________________________________________________________________________
___________________________________________________________________________________
________________________________________________________________________________

4. Cluster Sampling
___________________________________________________________________________________
___________________________________________________________________________________
________________________________________________________________________________

Reflection:
1. What are your learnings from the activites?
___________________________________________________________________________________
___________________________________________________________________________________
________________________________________________________________________________

2. What part of the lesson did you like most?


___________________________________________________________________________________
___________________________________________________________________________________
________________________________________________________________________________

3. What challenges did you encounter from the activities?

62
___________________________________________________________________________________
___________________________________________________________________________________
________________________________________________________________________________

References:

Belecina, Rene R., Elisa S. Baccay, and Efren B. Mateo (2016). Statistics and Probability, Manila: Rex Book
Store Inc., 101-131

Bluman, A.G. (2012). Elementary Statistics: A Step-by-Step Approach, 8th Ed. The McGraw-Hill Companies, Inc.

Febri, Francisco Jr. (1987). Introduction to Statistics. Quezon City: Phoenix Publishing House, Inc. 85-101

https://faculty.elgin.edu/dkernler/statistics/ch01/1-4.html

Prepared by:

ARIS B. NISOLA
Writer
Masarawag National High School

sampling
sampling
. Clustered random 4
sampling
. Simple random 3
.sampling
Systematic random 2
Me!
. Stratified random 1
Activity 2: Identify

. Random sample 4
. not 3
. not 2
Not?
. Random sample 1
Activity 1: Random Sample or
Key:
Answer

63
STATISTICS AND PROBABILITY

Name: __________________________________ Grade Level: _________________________


Section: _________________________________ Date: _______________________________

LEARNING ACTIVITY SHEET


DISTINGUISHING PARAMETER AND STATISTIC

Background Information for Learners

In the previous activity, you learned about random sampling, a process done in selecting an
unbiased representative sample from a population.
In this particular Learning Activity Sheet (LAS), you will be able to identify and distinguish measures
about population and sample called parameter and statistic. Take time to read the examples below
for you to be able to answer the succeeding activities.

Parameter and Statistic

A measure that describes a population is called a parameter. A measure that describes a

sample is called statistic. There is a simple and straightforward way to remember what a

parameter and statistic are measuring. All that we must do is look at the first letter of each word.

A parameter measures something in a population, and a statistic measures something in a sample.

Examples:

Parameter: the average IQ obtained from the entire population of grade 11 students

Statistic: the average IQ obtained from a sample of grade 11 students

Parameter: The average weekly allowance of all Grade 11 male students

Statistic: The average weekly allowance of a sample of 60 male students

Parameter: The average time spent by all the teachers at a certain school in sorting Self-Learning

64
Modules is 2.2 hours.

Statistic: A survey revealed that from the randomly selected sample of 400 high school students

in the Albay Division, 75% do not have internet connection at home.

In population parameter, mean is represented by μ, σ2 represents variance, and σ represents

standard deviation. In sample statistics, sample mean is represented by 𝑥̅, s2 represents sample

variance and s represents sample standard deviation.

Learning Competency

The learner distinguishes between parameter and statistic. (M11/12SP-IIIc-3)

ACTIVITY 1: TRUE OR FALSE?


Determine whether the statement is true or false. Write T if the statement is true and F if false on
the blank provided before each item.

____1. The given value in ”The average score of all the TVL students in Statistics and Probability of a certain
school is 83%.” is a parameter.
____2. An example of a parameter is s2.
____3. The descriptive measure computed from the entire population of data is called parameter.
____4. The average age of all the grade 11 students is an example of statistic.

ACTIVITY 2: PARAMETER OR STATISTIC?


Identify whether the given value is a parameter or a statistic.

_____________1. A researcher found out that the average salary of all the employees in a certain
company is Php 16,240.
_____________2. The average score of 36 out of 150 Grade 11 students in an examination is 46.
_____________3. Based on a sample of 400 students, it was found out that 78% of them do not have
gadgets for online learning.

65
_____________4. The teacher surveyed all 48 students in her advisory class about their preferred
distance learning delivery modality and found out that 83% of them wanted modular learning.

ACTIVITY 3: WHICH IS WHICH?


Read the statistical study in each item. For each study, identify both the parameter and the
statistic. Refer to the example below.

Example: A researcher wants to determine the average time spent by grade 11 students in reading
answering Statistics modules. From a random sample of 60 grade 11 students, he obtained a sample
mean time of 4.2 hours.

Parameter: The average time spent by all grade 11 students in reading and answering Statistics
modules
Statistic: The average time of 4.2 hours from a sample of 60 grade 11 students

1. A teacher wants to determine the average age of grade 11 students. From a random sample
of 40 students, the teacher obtained an average age of 17.
Parameter: ________________________________________________________________________
Statistic: __________________________________________________________________________

2. A teacher wants to know the average time spent in playing online games of his advisory class.
He surveyed a random sample of 30 students and found out that they spend an average of 4 hours
per day in playing online games.
Parameter: ________________________________________________________________________
Statistics: _________________________________________________________________________

3. A researcher wants to determine the average time spent by Statistics teachers in preparing
learning activity sheets. She randomly selected 50 teachers and obtained an average time of 2.5
hours.
Parameter: ________________________________________________________________________
Statistic: __________________________________________________________________________

4. A researcher wants to determine the mean height of grade 11 male students. He obtained a
mean height of 154 cm from the 60 male students he randomly selected.

66
Parameter: ________________________________________________________________________
Statistic: __________________________________________________________________________
Reflection:
1. What have you learned from this activity?
__________________________________________________________________________________
__________________________________________________________________________________
__________________________________________________________________________________

2. What part of the lesson did you like most?


__________________________________________________________________________________
__________________________________________________________________________________
__________________________________________________________________________________

3. What challenges did you encounter from the activities?


__________________________________________________________________________________
__________________________________________________________________________________
__________________________________________________________________________________

References:

Belecina, Rene R., Elisa S. Baccay, and Efren B. Mateo (2016). Statistics and Probability, Manila: Rex
Book Store Inc., 101-131

Bluman, A.G. (2012). Elementary Statistics: A Step-by-Step Approach, 8th Ed. The McGraw-Hill
Companies, Inc.

Febri, Francisco Jr. (1987). Introduction to Statistics. Quezon City: Phoenix Publishing House, Inc.
85101

https://faculty.elgin.edu/dkernler/statistics/ch01/1-4.html

Prepared by:

ARIS B. NISOLA
Writer
Masarawag National High School

67
68
Grade Level: _________________________ Name: __________________________________
STATISTICS AND PROBABILITY
Answer Key:
Activity True
1: or False?
1 .T
2 .F
3 .T
4 .F
Activity Paramter
2: or Statistic?
1 . Parameter
2 . Statistic
3 . Parameter
4 . Statistic
Activity 3: Which is which?
1 . Parameter:
The average age of all grade 11 students
Statistic:
The average age of 17 from a random sample of 40 students
2 . Parameter:
The average time spent by all his students in playing online games
Statistic:
The average time of 4 hours per day spent by a random sample of 30 students in
playing online games
3 . Parameter:
The average time spent by all Statistics teachers in preparing learning activity
sheets
Statistic:
The average time of 2.5 hours spent by 50 randomly selected teachers in preparing
learning activity sheets
4 . Parameter:
The mean height ofrade
all g11 male students
Statistic:
The mean height of 154 cm of the 60 randomly selected grade 11 male students
Section: _________________________________ Date: _______________________________

LEARNING ACTIVITY SHEET


IDENTIFYING SAMPLING DISTRIBUTIONS OF STATISTICS (SAMPLE MEAN)

Background Information for Learners


You learned in the previous chapters about discrete probability distribution and continuous
probability distribution. In this activity, you will learn how to identify and construct sampling
distribution of the sample means. Are you ready?
Take time to read the discussion and procedure on how to construct sampling distribution of the
sample means.

Warm-Up

One of the prerequisite skills that you need to have in this activity is for you to compute for the mean of a
given set of numbers. Can you find the mean of the sets of numbers below?
1) 2, 4, 7, 9 2) 3, 10, 5, 8, 4 3) 9, 10, 12, 15, 18, 20

If your answers are: 5.5, 6 and 14 for items 1, 2 and 3 respectively, then you did a great job in finding the
mean of the given set of numbers!

List Them All!

Suppose a population consists of these numbers: 2, 4, 8 and 10. Can you list all the possible

samples of size 2 which can be drawn from this population? One possible sample is shown below.

Sample

2, 4

Were you able to identify all samples of size 2? Check your answers by referring to the list
below.
Sample
2, 4
2, 8
2, 10
4, 8
69
4, 10
8, 10

Mean-ie Me!

Now that you were able to list all the possible samples of size 2 from the population
containing the numbers 2, 4, 8 and 10, it’s time to compute for the mean of each sample.

Sample Mean
2, 4 3
2, 8 5
2, 10 6
4, 8 6
4, 10 7
8, 10 9

This let us learn how to make a probability distribution of the sample means, which is
called the sampling distribution of sample means. Study the table below.

Sample Mean Probability


Frequency
𝒙̅ P(𝒙̅)
1
3 1
6
1
5 1
6
1
6 2
3
1
7 1
6
1
9 1
6

Notice that the first column contains all the possible sample means. The second column contains the
frequency of each sample mean. Take note that in the previous activity, two samples of size 2 have a
mean of 6. Finally the third column contains the probabilities of the possible sample means.

70
The probability distribution of the sample means is also called the sampling distribution of the
sample means. This describes the probability for each mean of all samples with the same sample size
n.

Learning Competency
The learner identifies sampling distributions of statistics (sample mean) – M11/12SP-IIIc-4

ACTIVITY 1: LIST AND CONSTRUCT!

A population consists of the five numbers 2, 5, 6, 8, and 11. Consider samples of size 2 that can be
drawn from the population.

a. List all the possible samples and the corresponding mean.

Sample Mean

b. Construct the sampling distribution of the sample means.

Sample Mean Probability


𝒙̅ Frequency P(𝒙̅)

71
ACTIVITY 2: TEST SCORES

A group of students got the following scores in a test: 6, 9, 12, 15, and 21. Consider samples of size 3
that can be drawn from this population.

a. List all the possible samples and the corresponding mean.

Sample Mean

b. Construct the sampling distribution of the sample means.

Sample Mean Probability


𝒙̅ Frequency P(𝒙̅)

72
ACTIVITY 3: DRAWING CARDS

Samples of 4 cards are drawn from a population of 6 cards numbered 1-6.

Construct a sampling distribution of the sample means and answer the following questions:

1. How many samples of size 4 can be drawn from the population?


2. What are the possible means?
3. What is the probability of getting 4 as a mean?
4. What is the probability of getting 3.5 as a mean?

(You may use an extra sheet of paper for this activity.)

Reflection:
1. What have you learned from this activity?
__________________________________________________________________________________
__________________________________________________________________________________
__________________________________________________________________________________

2. What part of the lesson did you like most?


__________________________________________________________________________________
__________________________________________________________________________________
__________________________________________________________________________________

3. What challenges did you encounter from the activities?


__________________________________________________________________________________
__________________________________________________________________________________
__________________________________________________________________________________

73
References:

Belecina, Rene R., Elisa S. Baccay, and Efren B. Mateo (2016). Statistics and Probability, Manila: Rex
Book Store Inc., 101-109

Bluman, A.G. (2012). Elementary Statistics: A Step-by-Step Approach, 8th Ed. The McGraw-Hill
Companies, Inc.

Febri, Francisco Jr. (1987). Introduction to Statistics. Quezon City: Phoenix Publishing House, Inc.
85101

https://faculty.elgin.edu/dkernler/statistics/ch01/1-4.html

Prepared by:

ARIS B. NISOLA
Writer
Masarawag National High School
4) 1/ 5 2.5, , 3, 3.25, 3.5, 3.75, 4, 4.25, and
4.5 2) 2.75
3) 1/ 5 15 1)
Answers to the questions:

4.5 1 1 / 15
4.25 1 1 / 15
4 3 1/ 5
3.75 1 1 / 15
3.5 3 1/ 5
3.25 2 2 / 15
3 2 2 / 15
2.75 1 1 / 15
2.5 1 1 / 15
𝒙
̅
Mean ̅ )
P(𝒙
Frequency
Sample Probability

Activity 3: Drawing Cards

74
STATISTICS AND PROBABILITY
Name : _____________________________________ Grade Level: _____________

Section: _____________ Date : __________________

LEARNING ACTIVITY SHEET


FINDING THE MEAN AND VARIANCE OF SAMPLING DISTRIBUTION OF THE SAMPLE MEANS

Background Information for Learners

Hello dear young statistician! You are now on the 6 th week of Quarter 3.
In the previous lesson you have learned several ways to select a random sample
from a population.

This week, we will be exploring the Mean and Variance of Sampling Distribution
of the Sample Means. In this lesson, you will learn how to describe the
sampling distribution of the sample means using the means and
variance.

Now, it’s time for you to explore and learn. Good luck!
• The probability distribution of the sample means is also called the sampling distribution of the sample
means.
75
• A sampling distribution of sample means is a frequency distribution that describes the probability for
each mean of all samples with the same sample size n.

• The standard deviation of the sampling distribution of the sample means is also known as the standard
error of the mean. It measures the degree of accuracy of the sample mean (x̄ ) as an estimate of the
population mean (µ).

• When taking samples of size n from any population with finite mean µ and variance σ2,then the sampling
distribution of the sample means has the following properties:
1. The mean of the sampling distribution of the sample means ( µx̄ ) is equal to the population
mean ( µ ).
That is, µx̄ = µ
2. The variance of the sampling distribution of the sample means ( σ2 x̄ ) is smaller than the
population distribution, which is given by, ✓ σ2 x̄ =
𝝈𝟐
( with replacement )
𝒏

✓ σ2 x̄ = 𝝈𝟐 • 𝑵− 𝒏 ( without replacement )
𝒏 𝑵− 𝟏

3. The standard deviation of the sampling distribution of the sample means is equal to the
population standard
deviation divided by the square root of the sample size of n of the samples. That is,
✓ ̄
𝛔
( with replacement )

✓ ̄ ( without replacement )

• Formula’s in Computing the Mean, Variance, and Standard Deviation


SAMPLING
POPULATION DISTRIBUTION OF
SAMPLE MEANS
∑ 𝐗 µ x̄ = ∑ 𝒙̄
MEAN µ =
𝐍
• 𝑷 ( 𝒙̄ )

σ2 x̄ =
VARIANCE σ2 = ∑
( 𝑿− µ )𝟐
∑ 𝐏(𝐱̄̄) • ( 𝐱̄̄ − µ )𝟐
𝑵
STANDAR σ x̄ =
σ = √
D
DEVIATION 𝑵 √∑ 𝐏(𝐱̄̄) • ( 𝐱̄̄ − µ )𝟐

76
In this learning activity sheet, you will learn how to describe the sampling distribution of the
sample means by computing its mean and variance.

Learning Competency
1. Find the Mean and Variance of the Sampling Distribution of the Sample Mean ( M11/12SP-IIId-5 )
2. Define the sampling distribution of the sample mean for normal population when the variance is (a)
known; and (b) unknown (M11/12SP-IIIe-1)

Example 1: SAMPLING without REPLACEMENT from a FINITE POPULATION

Consider a population consisting of the values 1, 2, 3, 4, and 5. Suppose samples of size 2 are drawn from
without replacement from the population. Describe the sampling distribution of the sample means.
a) Compute the mean, variance, and standard deviation of the population.
b) Find the mean, variance, and standard deviation of the sampling distribution of the sample means.

STEPS SOLUTION

1. Compute the mean of the population (µ). µ = = 𝟏+𝟐+ 𝟑+ 𝟒+ 𝟓 =


3.00
𝐍 𝟓
Hence, the mean of the population is 3.00

2. Compute the variance of the population (σ).


X X-µ (X-µ)2

1 -2 4
2 -1 1
3 0 0
4 1 1
5 2 4

∑( 𝑿 − µ ) 2 = 10

= 𝟏𝟎 = 2
𝑵 𝟓
Hence, the variance of the population is 2.

And the population standard deviation is 1.41.


Use the formula N C n where N = 5 and n = 2
3. Determine the number of possible
samples of size n = 2 5 C 2 = 10
Hence, there are 10 possible samples of size 2 that can
be drawn.

4. List all possible samples and their


Samples Mean
corresponding means.
1 ,2 1.50

77
1 ,3 2.00
1 ,4 2.50
1 ,5 3.00
2 ,3 2.50
2 ,4 3.00
2 ,5 3.50
3 ,4 3.50
3 ,5 4.00
4 ,5 4.50

5. Construct the sampling distribution of Sampling Distribution of Sample Means


the sample means.
Sample
Probability
Mean Frequency
P ( x̄ )

1.50 1

2.00 1

2.50 2

3.00 2

3.50 2

4.00 1

4.50 1

TOTAL 10 1.00

78
6. Compute the mean (µ x̄ ) of the sampling Sample Probabilit
distribution of the sample means. Mean y x̄ • P ( x̄ )
Follow these steps: x̄ P ( x̄ )

a. Multiply the sample mean by the 1.50 0.15


corresponding probability.

b. Add the results. 2.00 0.20

2.50 0.50

3.00 0.60

3.50 0.70

4.00 0.40

4.50 0.45

∑ 𝒙̄ • 𝑷 ( 𝒙̄ )
TOTAL 1.00
= 3.00

µ x̄ = ∑ 𝒙̄ • 𝑷 ( 𝒙̄ )
= 3.00

Hence, the mean of the sampling distribution


of the sample means is 3.00

7. Compute the variance (σ2 x̄ )of the sampling P ( x̄ ) x̄ - µ (x̄ - µ x̄ ) P ( x̄ ) • (x̄ -



distribution of the sample means . x̄ 2
µ x̄ ) 2
Follow these steps: 1.5 0
-1.50 2.25 0.225
a. Subtract the population mean (µ) from each
sample mean (x̄ ). Label this as x̄ - µ 2.0 0
-1.00 1.00 0.100
b. Square the difference. Label this as
( x̄ − µ ) 2 2.5 0
0.50 0.25 0.050
c. Multiply the results by the corresponding
probability. Label this as 3.0 0
0.00 0.00 0.000
P ( x̄ ) • ( x̄ − µ ) 2
d. Add the results. 3.5 0
0.50 0.25 0.050
4.0 0
1.00 1.00 0.100

4.5
1.50 2.25 0.225
0

79
TO
1.00 0.75
TAL

µ
σ2 x̄ = ∑ 𝐏(𝐱̄̄) • ( 𝐱̄̄ − 𝐱̄̄)𝟐
= 0.75

Hence, the variance of the sampling


distribution is 0.75.

σ x̄ = √∑ 𝐏(𝐱̄)̄ • ( 𝐱̄̄ − µ )𝟐
= 0.87
And the standard deviation of the sampling
distribution is 0.87.

Example 2: SAMPLING with REPLACEMENT from a FINITE POPULATION

Consider a population consisting of the values 2, 5, and 8. List all the possible samples of size n = 2
which can be drawn with replacement from the population. Describe the sampling distribution of
the sample means.
a) Compute the mean, variance, and standard deviation of the population.
b) Find the mean, variance, and standard deviation of the sampling distribution of the sample
means.
STEPS SOLUTION

1. Compute the µ =
population mean (µ 𝐍 𝟑
) Hence, the population mean is 5.00

2. Compute the
population X X-µ (X-µ)2
variance (σ2) and 2 -3 9
population
standard deviation 5 0 0
(σ). 8 3 9

∑( 𝑿 − µ ) 2 = 18

𝑵 𝟑
Hence, the population variance is 6.

And the population standard deviation is 2.45.

3. List all Mean


possible samples Samples

of size 2 ,2 2.0
2 , with

80
replacement and 2 ,5 3.5
their
2 ,8 5.0
corresponding
means. 5 ,2 3.5
5 ,5 5.0

5 ,8 6.5
8 ,2 5.0
8 ,5 6.5
8 ,8 8.0

Sampling Distribution of Sample Means


4. Construct the Sample Mean x̄ Probability
sampling Frequency
P ( x̄ )
distribution of
the sample 2.00 1
means.
3.50 2

5.00 3

6.50 2

8.00 1
TOTAL 9 1.00

5. Compute the Sample Mean Probability µ


mean (µ x̄ ) of the x̄ P ( x̄ ) x̄ • P ( x̄ ) x̄ =
sampling
distribution of
2.00 0.22
the sample
means.
3.50 0.78

5.00 1.67

6.50 1.44
81
8.00 0.89
TOTAL 1.00 ∑ 𝒙̄ • 𝑷 ( 𝒙̄ ) = 5
∑ 𝒙̄ • 𝑷 ( 𝒙̄ )
= 5.00

Hence, the mean of the sampling distribution of the sample


means is 5.

6. Compute the P ( x̄ ) • (x̄ -


x̄ P ( x̄ ) x̄ - µ (x̄ - µ x̄ ) 2
variance (σ2 x̄ ) and x̄ µ x̄ ) 2
the
standard
deviation (σ x̄ ) of 2.00 -3.00 9.00 1.00
the sampling
distribution of 3.50 -1.50 2.25 0.50
the sample
means.
5.00 0.00 0.00 0.00

6.50 1.50 2.25 0.50

8.00 3.00 9.00 1.00

TOTAL 1.00 3

µ
σ2 x̄ = ∑ 𝐏(𝐱̄̄) • ( 𝐱̄̄ − )𝟐
𝐱̄̄

= 3

Hence, the variance of the sampling distribution is 3.

σ x̄ = √∑ 𝐏(𝐱̄)̄ • ( 𝐱̄̄ − µ )𝟐

= 1.73
And the standard deviation of the sampling distribution is 1.73.

Let us summarize what we have done for the preceding examples by comparing the means and
variances of the population and the sampling distribution of the means.

In EXAMPLE 1 In EXAMPLE 2

Population Sampling Population Sampling


(N=5) Distribution of (N=3) Distribution of
the Sample the Sample
82
Means Means
(n=2) (n=2)

MEAN µ = 3 µx̄ = 3 µ = 5 µx̄ = 5

VARIANCE σ2 = 2 σ2 x̄ = 0.75 σ2 = 6 σ2 x̄ = 3

STANDARD
σ = 1.41 σ2 x̄ = 0.87 σ = 2.45 σ2 x̄ = 1.73
DEVIATION

NOTE:

Observe that the mean of the sampling distribution of the sample means is always equal to the mean
of the population. And the variance of the sampling distribution of the sample means is smaller
than the population distribution

Example 3: SAMPLING DISTRIBUTION OF THE SAMPLE MEANS FROM AN INFINITE


POPULATION

A population has a mean of 60 and a standard deviation of 5. A random sample of 16 measurements is


drawn from this population. Describe the sampling distribution of the sample means by computing its
mean and standard deviation. We shall assume that the population is infinite.
STEPS SOLUTION

1.Identify the given information. Here µ = 60 , σ = 5 , and n = 16

µx̄ = µ
2. Find the mean of the sampling distribution. = 60
Use the
property that µx̄ = µ Hence, the mean of the sampling
distribution of the sample means is 60.
σ x̄ =
3. Find the standard deviation of the sampling 𝛔
𝛔
distribution. Use the property that σ x̄
𝟓

𝟓
= or 1.25
𝟒

Hence the standard deviation of the sampling


distribution is 1.25.

Example 4: Sampling Distribution of the Sample Means for Normal Population When the Variance is
Unknown

A sample of size 9 is drawn from a normal population having a mean µ = 8 and the
standard deviation σ = 5. Suppose that the resulting sample is 11, 5, 7, 11, 10, 13, 13, 11, 9.
1. What is the sampling error based on this sample?

83
2. Determine the mean and standard deviation of the sampling distribution of sample means for samples
having the same size.

SOLUTION:
1. Using the provided data, the sample mean can be computed as follows:
x̄ = 𝟏𝟏 + 𝟓 + 𝟕 + 𝟏𝟏 + 𝟏𝟎 + 𝟏𝟑 + 𝟏𝟑 + 𝟏𝟏 + 𝟗 = 10
𝟗

The sampling error is the difference between this value and the corresponding population
mean,

that is, 10 – 8 = 2
2. Our population has mean µ = 8 and and standard deviation σ = 5. Using the previous
generalization, the

sampling distribution of the sample means of samples with size 9 has a mean µx̄ = µ = 8 and a
standard
𝛔 𝟓 𝟓
deviation =
𝟑

Example 5: Sampling Distribution of the Sample Means for Normal Population When the Variance is
Known

A sample is drawn from a normal population with µ = 78.3 and σ = 5.6. How is the variance of the
sample mean affected when the sample size is
1. increased from 36 to 64 ?
2. decreased from 225 to 144 ?

SOLUTION: Since the population has standard deviation σ = 5.6, its variance is σ2 = ( 5.62 )2 = 31.36

1. The variance of the sample mean is decreased from


2. The variance of the sample mean is decreased from

ACTIVITY 1 : FIGURE IT OUT!


Direction: Choose the letter that corresponds to your answer.
________ 1. The weekly allowance of all the students in Malabog National High School has an
average of Php 50. and the standard deviation of Php 5. A random sample of 100 students was
asked their
weekly allowance.
What is the standard error of the sampling distribution of the mean weekly allowance
of students in this school?

a. 100 b. 5 c. 50 d.
________ 2. For the scenario in item 2, the mean of the sampling distribution of the mean
weekly allowance of the students is ________ .

a. 100 b. 5 c. 50 d.
________ 3. Suppose the sampling error is zero. Which of the following must be true?
a. There is an error in the computation of the sample mean.
b. The sample statistic and the population parameter are proportional.
84
c. The sample statistic and the population parameter are the same.
d. None of the above.
________ 4. If a population has a mean of 5.7, what is the mean of the sampling distributions of its
means?
a. -5.7 b. 5.7 c. 0 d. Cannot be
Determined

________ 5. How many possible samples of size 2 are there from the population consisting of the
values 10, 20, 40, and
70, if the sample is to be taken without replacement?
a. 4 b. 6 c. 8 d. 12

ACTIVITY 2 : COMPLETE & DESCRIBE ME!


Direction: Describe the sampling distribution of the sample means of scores in
the Summative Test in Statistics. Complete the table below and compute for the
mean, variance, and standard deviation.

(x̄ - µ x̄ )
x̄ P ( x̄ ) x̄ • P ( x̄ ) x̄ - µ x̄ 2
P ( x̄ ) • (x̄ - µ x̄ ) 2

12

15

µ
∑ 𝒙̄ • 𝑷 ( 𝒙̄ ) = ∑ P(x̄ ) • ( x̄ − x̄ )2 =
_____ ________

MEAN ( µ x̄ ) VARIANCE ( σ2 x̄ ) STANDARD DEVIATION ( σ x̄


)

References for Learners

Bataller, Ramil T. (2016). Statistics and Probability. Manila: Salesiana Books by Don Bosco Press,Inc.

Belecina, R., Baccay, E., & Mateo, E. (2016). Statistics and Probability. Manila: Rex Book Store, Inc.

Reyes, Maria Angeli T. (2017). Statistics and Probability. Manila: C & E Publishing, Inc.

https://www.slideshare.net/sirgibey/cabt-shs-statistics-probability-mean-and-variance-ofsampling-
distributions-of-

85
samplemeans?from_m_app=android&fbclid=IwAR1Sm1liVb1v2wqFmJUZim7WGQbTzCSQ1waKZIU
npmxKagLT3FcfMc3Z8ls

86
STATISTICS AND PROBABILITY

Name: ____________________________________________ Grade: ___________________ Section:


___________________________________________ Date: ____________________

LEARNING ACTIVITY SHEET

SAMPLING DISTRIBUTION OF THE SAMPLE MEAN USING CENTRAL LIMIT THEOREM

Background Information for Learners

• The Central Limit Theorem states that as the sample size n increases without limit, the shape
of the distribution of the sample means taken with replacement from a population with
mean µ and standard deviation 𝝈 will approach a normal distribution. This distribution will
have a mean µ and a standard deviation 𝛔/ .
• A sampling distribution of sample means is a distribution using the means computed from all possible
random samples of a specific size taken from a population.

• Properties of the Distribution of Sample Means


1. The mean of the sample means (𝜇𝑥̅) will be the same as the population mean (µ).
2. The standard deviation of the sample means will be smaller than the standard deviation of
the population(𝜎𝑥̅ < 𝜎), and it will be equal to the population standard deviation divided by
the square root of the sample size ( 𝜎𝑥̅ =).

Learning Competencies
• Illustrates the Central Limit Theorem: M11/12SP-IIIe-2
• Defines the sampling distribution of the sample mean using the Central Limit Theorem:
M11/12SP-III-3

Activity 1. Let’s Discover

Read the situation and answer the given questions.


Suppose a teacher gave a 4-item quiz to all of his/her class with a total of four hundred
students. Assumed that each correct answer was given 2 points and no one in the class got a zero
score. After recording the teacher found out that 100 students got 2 points, 100 students got 4
points, 100 students got 6 points, and another 100 students got 8 points.

1. What is the mean (𝜇) score of the class?


a. 2 b. 3 c. 4 d. 5

2. What is the Standard Deviation of the students’ score? Use the formula below.

87
𝜎

a. 2 .236 b. 3 c. 1.581 d. 5

3. If you make a histogram of the students’ score what would it look like?

a b c d

Suppose that the teacher who gave the 4-item quiz is teaching you statistics and he/she
wants you to take the mean and the standard deviation of the class in a short period of time without
giving you the summary of the result. Since you are in a hurry you just took few samples out of quiz
scores. Now, if all samples of size 2 are taken with replacement and the mean of each sample is
found as shown below.
Sample Mean Sample Mean
2,2 2 6,2 4
2,4 3 6,4 5
2,6 4 6,6 6
2,8 5 6,8 7
4,2 3 8,2 5
4,4 4 8,4 6
4,6 5 8,6 7
4,8 6 8,8 8

4. What is the mean of the sample mean?


a. 2 b. 3 c. 4 d. 5

5. What is the standard deviation of the sample mean?


a. 2 .236 b. 3 c. 1.581 d. 5

6. The mean of the population is _____ to the mean of the sample mean
a. not equal c. equal
b. greater than d. less than

7. The population standard deviation is ______ to the standard deviation of the sample mean
a. not equal c. equal
b. greater than d. less than

8. Based on the figure in activity 1 the dispersion (width) of the sample mean ______ is than
the population distribution
a. shorter c. wider
b. taller d. narrower

88
9. If a population is not normally distributed, the distribution of the sample means for a given
sample size n will ____________. a. be positively skewed
b. be negatively skewed
c. takes the same shape as the population
d. approach a normal distribution as n increases

10. The standard deviation of the sampling distribution of the sample means according to the
Central Limit Theorem is
a. exactly equal to the standard deviation
b. close to the population standard deviation if the sample size is large
c. equal to the population standard deviation divided by the square root of the sample size
d. cannot be determined

Activity 2. LOOK AT ME

Observe the given figure and answer the given questions

NOTE: the line in the middle of each graph represent the mean of the population.
Population A B C D

Histogram of the
Population

Histogram of the
Sampling Distribution
when the sample size is
equal to 2
Histogram of the
Sampling Distribution
when the sample size is
equal to 6

Histogram of the
Sampling Distribution
when the sample size is
equal to 30

The figure above is based from the page 227 of the book Basic Statistics for Business and Economics by D.A. Lind,
W.G. Marchal, and S.A. Wathen (McGraw-Hill, 2006)

1. The graph of Population A is ____________.


a. Symmetric, Unimodal c. Symmetric, Bimodal
b. Uniform d. Rectangular

2. The graph of Population C is ____________.


a. Symmetric, Unimodal c. Symmetric, Bimodal
b. Uniform d. Rectangular
89
3. The shape of a Normal Distribution is like a ________.
a. Ghost c. Rectangle
b. Bell d. Triangle

4. Based on the figure as the value of sample size (n) gets ______, the curves approach a
normal distribution.
a. Smaller c. nearer to zero
b. Larger d. fatter

5. The graph population A and Population C are both symmetrical (but not normal). At what
value of n did the shape of the sampling distribution looks normal?
a. 2 b. 6 c. 4 d. 30

6. Population D is skewed to the right, at what value of n did the shape of the sampling
distribution looks normal?
a. 4 b. 6 c. 30 d. 2

7. At what value of n did all of the graph of the sampling distribution look similar?
a. 2 b. 6 c. 4 d. 30

8 - 10
We can conclude that regardless of the initial __(8)__ of the population distribution, if
samples of size n are randomly selected from a population, the sampling distribution of the
sampling means will __(9)__ a normal distribution as the sample size n gets__(10)__ .
a. Shape c. Equal to
b. approach d. larger

90
Reflection
What is the importance of the use of CLT in our daily life?

________________________________________________________________________________
_____________________________________________________________________________

Why is it useful in solving problems involving sampling?


________________________________________________________________________________
_____________________________________________________________________________

References

Bluman, A. G. (2009). Elementary Statistics A Step by Step Approach. New York: McGraw-Hill.

Chan Shio, C. O., & Reyes, M. T. (2017). Statistics & Probability for Senior High School. Quezon City: C & E
Publishing, Inc.
Lind, D. A., Marchal, W. G., & Wathen, S. A. (2006). Basic Statistics for Business and Economics. New York:
McGraw-Hill.

Prepared by:

91
Giovani G. Naag, ABE
Special Science Teacher I
Malabog National High School

92
10 . C
9 .D
8 .D
7 .B
6 .C
5 .C
4 .D
3 .B
2 .A
1 .D
Activity 2.

.D 10
.B 9
.A 8
.D 7
.C 6
.C 5
.B 4
.B 3
.B 2
.C 1
Activity 1.

Answer Key

STATISTICS AND PROBABILITY

Name of Learner: _________________________________ Grade Level: ____________

Section: __________________________________________ Date: __________________

LEARNING ACTIVITY SHEET

SOLVING PROBLEMS INVOLVING SAMPLING DISTRIBUTIONS OF THE SAMPLE MEAN

BACKGROUND INFORMATION FOR LEARNERS

A theoretical probability distribution of sample means that would be obtained by drawing from the
population all possible samples of the same size. In other words, the distribution of all possible values
from all the random samples of given size n .

93
 The probability distribution of sample mean (hereafter, will be denoted as 𝑋̅) is called the
sampling distribution of the mean (also, referred to as the distribution of sample mean).
 Like 𝑋̅, we call sampling distribution of variance (denoted as 𝑆2).
 Using the values of 𝑋̅ and 𝑆2 for different random samples of a population, we are to make
inference on the parameters 𝜇 and 𝜎2 (of the population).

Population parameter: a numerical descriptive measure of a population.

(for example: μ, 𝝈, p (a population proportion); the numerical value of a population parameter is usually
not known)

Examples: μ = mean height of all students

p=proportion of residents who favor stricter curfew p ̂ (or p-

hat)=the mean of all sample proportions

standard deviation=

Sample statistic: a numerical descriptive measure calculated from sample data. (e.g, x, s, ̄ p̂(sample
proportion))

Example 1 of sampling distribution of p, the sample proportion ̂

• If a coin is fair the probability of a head on any toss of the coin is p = 0.5 (p is the
population parameter)
• Imagine tossing this fair coin 4 times and calculating the proportion p̂ of the 4 tosses that
result in heads (note that p̂ = x/4, where x is the number of heads in 4 tosses).

• Objective: determine the sampling distribution of p, the proportion of heads in 4 tosses of ̂


a fair coin.

0 1 2 3 4
Possible values of :pˆ = 0, =.25, =.50, =.75, =1

4 4 4 4 4
4
There are 2 = 16 equally likely possible outcomes

(1 =head, 0 =tail)

(1,1,1,1) (1,1,1,0) (1,1,0,1) (1,0,1,1)

(0,1,1,1) (1,1,0,0) (1,0,1,0) (1,0,0,1)

(0,1,1,0) (0,1,0,1) (0,0,1,1) (1,0,0,0)


94
(0,1,0,0) (0,0,1,0) (0,0,0,1) (0,0,0,0)

p̂ 0.0 0.25 0.50 0.75


(0 heads) (1 head) (2 heads) (3 heads)

P(p) 1/16= 4/16= 6/16= 4/16=


0.0625 0.25 0.375 0.25

Sampling Distribution of p-hat

Values of p-hat

Learning Competency
• Solve problems involving sampling distributions of the sample mean. M11SP/IIIe-f-1

Activity 1 LET’S PLAY!

Consider five identical balls numbered and weighting as 1, 2, 3, 4


and 5. Consider an experiment consisting of drawing two balls,
replacing the first before drawing the second, and then computing the
mean of the values of the two balls.

1. Following the given set of data, complete the table below by


providing their means and mass functions.
2. Draw the graph.

𝑿̅

𝒇(𝑿̅)

Activity 2 ONE, TWO, THREE, VOTE!

1. A school polling organization polls 100 randomly selected registered student voters to estimate the
proportion of a large population that intends to vote for Candidate X in an upcoming election.
95
Although it is not known by the polling organization, the actual proportion of the population that prefers
Candidate X is .55.

(a) Give the numerical value of the mean of the


sampling distribution of pˆ.

(b) Calculate the standard deviation of the sampling


distribution of pˆ.

(c) If the proportion of the population that prefers


Candidate X is .55, would a sample proportion value of .70
be considered unusual?

Reflection

1. What have you learned from the lesson?


___________________________________________________________________________

__________________________________________________________________________________
_________________________________________________________________________________
_

2. Why do we need to apply sampling distribution in most of our daily tasks?


___________________________________________________________________________

__________________________________________________________________________________

__________________________________________________________________________________

3. What is the role of sampling distribution in solving both simple and complex word
problems?
___________________________________________________________________________

__________________________________________________________________________________

__________________________________________________________________________________
References

• The Sampling Distribution of the Sample Mean. (2021, February 25). Retrieved March 12, 2021,
from https://chem.libretexts.org/@go/page/570
• Lind, D.A., Marchal, W.G. and Wathen, S.A. Basic Statistics for Business and Economics. 5th ed.
McGraw-Hill. 2006.
• Bluman, A. G. Elementary Statistics step by step approach. 7th Ed. McGraw-Hill. 2009.

96
Prepared by

Albert S. Corbilla
Writer
San Agustin Integrated School

97
98
STATISTICS AND PROBABILITY

Name : ____________________________________ Grade Level : _________________________


Section : ___________________________________ Date : _______________________________

LEARNING ACTIVITY SHEET #9-1


ILLUSTRATING T-DISTRIBUTION

Background Information for Learners t- distribution which is also known as Student’s t-


distribution, is a probability
distribution which is utilized in estimating parameters of a certain population in case of the
sample size is small and/or the population variance or standard deviation is unknown. It is
like with the z-distribution that is bell-shaped and symmetric about the x- axis but flatter and
more spread.
X

Take note that z-test is only used when the population standard deviation or
variance is known and/or the sample size is large enough. But for sample sizes which are small,
sometimes the population standard deviation or variance is unknown, rely on the t-distribution
whose value can be known using:

𝑥̅ − 𝜇
𝑡= 𝑠

where 𝑥 is the sample mean, 𝜇 is the population mean, s is the standard deviation of the sample
and n is the sample size.
In addition, there is a lot of different or family of t-distributions. Its particular form is
determined
by its degrees of freedom. Degrees of freedom refers to the number of independent
observations in a given set of data. The number of independent observations is
sample size minus one or in symbols, that is df= n-1 where df is the degrees of freedom
99
and n is the sample size. Hence, given a sample size of 9 will have a degrees of freedom 8 and if
given a degrees of freedom of 14 will mean a sample size of 15. However, for some or other
applications, degrees of freedom can be calculated in a different way.

Moreover, some properties of t-distribution is that the mean distribution is equal to 0


and its variance is greater than 1 although close to 1. T- distribution is the same as the standard
normal distribution given with infinite degrees of freedom.
With any statistic that have a bell- shaped distribution with is approximately
normal, the t-distribution can be used. This means that the population distribution must be
normal, symmetric, unimodal and no outliers. Also, the t-distribution can still be utilized
for moderately skewed population distribution given that it is unimodal, without outliers and
size is at least 40. Lastly, the t-distribution can be applied for a size greater than 40 and without
outliers.
The t- distribution Table

100
In order to get the t- value, a t-distribution table is shown above which is consist of the
degrees of freedom (df) which is the numbers at the leftmost column, “α” which is some of the
special areas at the topmost row and the t-values which are located at the right of the degrees
of freedom and below “α”.
Identifying the t-value

To denote the t-value, it is conveniently written as (𝛼 ,𝑑𝑓). So, if you are ask on the tvalue
denoted by (0.05 ,8) means the t-value corresponding to 𝛼 = 0.05 and df= 8. To look for
this value in the t-table, first locate the 𝛼 = 0.05 on the top row and then the df on the

101
leftmost column. The intersection of 𝛼 = 0.05 and df= 8 is 1.860. (see illustration
below)

df 𝛼 0.1 0.05
1 3.078 6.314
2 1.886 2.920
: :

8 1.397
1.860

Learning Competency

The learner illustrates t- distribution (M11/12SP-IIIg-2).

Directions

Determine the process of finding t-value by looking at examples. Answer the What I can Do part.

Exercises
Activity 1
Determine what is asked based on the given data below.

1. n= 14 𝛼 = 0.1 (𝛼 ,𝑑𝑓)=

2. df= 10 𝛼 = 0.025 (𝛼 ,𝑑𝑓)=

3. (𝛼 ,𝑑𝑓)= 2.052 𝛼=0.025 n=

4. Find the t-value when 𝜇 = 42, 𝑥 = 44, s= 5 and n=25.

102
(Discussion of Activity 1)

Solution

1. n= 14 𝛼 = 0.1 ( 𝛼 , 𝑑𝑓 ) = 1.350

If n= 14, so df = n-1= 14-1= 13

df 𝛼

0.1 3.078

1 1.886

2 ⋮


13 1.350
1

2. df= 10 𝛼 = 0.025 ( 𝛼 , 𝑑𝑓 ) =2.228

df 𝛼 0.1 0.05 0.025

1 3.078 6.314

2 1.886 2.920

⋮ ⋮ ⋮

10 1.372 1.812

103
3. (𝛼 ,𝑑𝑓)= 2.052 𝛼=0.025 n= 28

df 𝛼 0.1 0.05 0.025

1 3.078 6.314

2 1.886 2.920

⋮ ⋮ ⋮

27
1.372 1.812

Since: df= 27 df= n –


1

27= n – 1
27+1= n (Addition Property of Equality)

n= 28

4. Find the t-value when 𝜇 = 42, 𝑥 = 44, s= 5 and n=25.


𝑥̅ − 𝜇 44 − 42 3 3
(α , df) = 𝑠 = 5
= = =3
5 1
√𝑛 √25 5

Guide Questions (if necessary)


1. When are you going to use the t-distribution?

2. What are the properties of t-distribution?

What can I do (Application)

Determine what is asked based on the given data below.

Determine what is asked based on the given data below.

1. n= 20 𝛼 = 0.005 (∝, 𝑑𝑓) = 2.

df= 28 𝛼 = 0.01 (∝, 𝑑𝑓) =

3. Find the t-value when 𝛼= 18.5, 𝜇 = 19, s= 2.5 and n=16.


104
Rubric for scoring

DISTINGUISHE PROFICIEN APPRENTIC NOVICE 1


D4 T3 E2

Uses Uses a Uses a Uses a Uses a


Representation representation that representation representation representatio n
s is unusual in its that clearly that gives some that gives little
mathematical depicts the important or no
precision problem information significant
about the information
problem about the
problem

Answers the Correct solution of Correct solution Copying error, No answer or


Problem problem and made a computational wrong answer
general rule about error, partial based upon an
the solution or answer for inappropriate
extended the problem with plan
solution to a more multiple
complicated solution answers, no
answer
statement,
answer labeled
incorrectly

Reflection t-Distribution value enables one to determine its location in the bell-shaped distribution
and
use as basis for making decisions. How do you find the t-value? When do you use t-value?
What about z-value?

References
Cervantes, Irl John M. Q3-Module 4-Estimation of Parameters, pp 1-8

Answer Key
2. 2.467

1. 1.729

Prepared by

HILDA C. REMENDADO
MT-II, PGCHS

STATISTICS AND PROBABILITY


105
Name : ____________________________________ Grade Level : _________________________
Section : ___________________________________ Date : _______________________________

LEARNING ACTIVITY SHEET #9-2

LEARNING ACTIVITY SHEET


IDENTIFYING PERCENTILE RANK AND t-DISTRIBUTION

Background Information for Learners


Percentile is one of the measurements in statistics which tells the value below in which an
observations’ percentage in a set of observations falls. For example, you score 70 in an exam and it
was mentioned that you the 85th percentile, it means that 85% of the scores are below you and 15%
of the scores are above you. In addition, in finding the percentile for a t- distribution, t-table can be
utilized as it is a number on a statistical distribution whose less- than the probability is the given
percentage. So, if you are ask on the 80th percentile of the t-distribution with respect to its degrees
of freedom, that refers to the value whose left tail is less than probability is 80% or 0.8 and whose
right tail or greater than probability is 20% or 0.2

Learning Competency with code


Identifying percentiles using t-distribution (M11/12SP-IIIg-5)

Directions
Study the solutions of exercises below, answer Application part by showing your solution in
a separate sheet and submit to your teacher.

Exercises
Activity 1 Determine what is asked based on the given data below.
1. n= 14 Percentile= 95th 𝑡(𝛼,𝑑𝑓)=
2. df= 10 Percentile= 90th 𝑡(𝛼,𝑑𝑓)=

Solution
1. n= 14 Percentile= 95th 𝑡(𝛼,𝑑𝑓)= 1.771 To find the value of 95th percentile, identify first the
degrees of freedom. df= n-1 = 14-1 = 13 To solve for 95th percentile, we need to understand first its
implication which is to get the t-value that is less than the probability 0.95 and the right tail
probability that is 0.05. Based on the table below, the 95th percentile is 1.771. df 𝛼 0.05 1 6.314 2
2.920 ⋮ ⋮ 13 1.771
2. df= 10 Percentile= 90th 𝑡(𝛼,𝑑𝑓)= 1.372 Since the degrees of freedom is given already, we
can now identify the 90th percentile which is 1.372. df 𝛼 0.1 1 3.078 2 1.886 ⋮ ⋮ 10 1.372

Application
Determine what is asked based on the given data below. Show solutions in a separate sheet of
paper.
1. n= 15 Percentile= 99.5th 𝑡(𝛼,𝑑𝑓)=
2. df= 25 Percentile= 97.5th 𝑡(𝛼,𝑑𝑓)=
3. n= 20 Percentile= 99.5th 𝑡(𝛼,𝑑𝑓)=
4. df= 31 Percentile= 97.5th 𝑡(𝛼,𝑑𝑓)=
5. n= 5 Percentile= 90th 𝑡(𝛼,𝑑𝑓)=

106
Guide Questions
1. Why is percentile important?
2. What is/are the disadvantages of percentile?
Rubric for scoring

DISTINGUISHE PROFICIEN APPRENTIC NOVICE 1


D4 T3 E2

Uses Uses a Uses a Uses a Uses a


Representation representation that representation representation representatio n
s is unusual in its that clearly that gives some that gives little
mathematical depicts the important or no
precision problem information significant
about the information
problem about the
problem

Answers the Correct solution of Correct solution Copying error, No answer or


Problem problem and made a computational wrong answer
general rule about error, partial based upon
the solution or answer for an
extended the problem with inappropriate
solution to a more multiple plan
complicated solution answers, no
answer
statement,
answer labeled
incorrectly

Reflection
Your percentile rank is your location in an area where you are above or at the right. Strive
more to be in higher spot for self-improvement. Use statistics to compute your percentile rank.
What application or impact does it bring your life?

References for learners


Cervantes, Irl John M. Q3-Module 4-Estimation of Parameters, pp 1-8

Answer Key 1.533 5.


1.96 4.
2.861 3.
2.060 2.
2.997 1.

107
Prepared by
HILDA C. REMENDADO
MT-II
STATISTICS AND PROBABILITY

Name of Learner: ____________________________ Grade Level: ____________


Section: __________________________________ Date: __________________

LEARNING ACTIVITY SHEET


COMPUTING THE LENGTH OF CONFIDENCE INTERVAL

BACKGROUND INFORMATION FOR LEARNERS

• A confidence interval is a specific interval estimate of a parameter determined by using data


obtained from a sample and by using the specific confidence level of the estimate. It is
simply a way to measure how well your sample represents the population you are studying.
• The confidence level is the probability that the confidence interval includes the true mean
value within a population.
𝐶𝑜𝑛𝑓𝑖𝑑𝑒𝑛𝑐𝑒 𝑙𝑒𝑣𝑒𝑙 = (1 − 𝑎)100%
• Length of Confidence Interval refers to the absolute difference between the upper
confidence limit and lower confidence limit.

108
IMPORTANT FORMULAS:
o
Given the upper and lower confidence limit
𝐿𝐶𝐼 = |𝑈𝐶𝐿 − 𝐿𝐶𝐿| = |𝐿𝐶𝐿 − 𝑈𝐶𝐿|
𝑳𝑪𝑰 = 𝑼𝑪𝑳 − 𝑳𝑪𝑳
o
Confidence interval of population mean (µ) with known variance,
-distribution table)
if n≥30 (use the z

𝑋 𝑍𝑎⁄
𝜎 𝜎
𝑍𝑎⁄
̅

𝝈
𝒁𝒂⁄𝟐
𝑳𝑪𝑰𝟐
o
Confidence interval of population mean (µ) with known variance, if
n<30 (use the t-distribution table)
𝜎 𝜎
𝑋̅
𝝈
𝒕𝒂⁄𝟐 𝑳𝑪𝑰𝟐

Where:

LCI- length of confidence interval

UCI- upper confidence limit

LCL- lower confidence limit


𝑍𝑎
⁄2 = z value from the standard normal distribution
curve
𝑡𝑎
⁄2 = t value with df=n-1
σ = standard deviation n=
sample size
Table1: Z-Scores for Commonly used Confidence Level
DESIRED CONFIDENCE LEVEL (Α) 𝒁𝜶⁄𝟐
90% 1.65
95% 1.96
99% 2.58

In this learning activity sheet, the learners will be able to know how to identify and compute for the
length of confidence interval.

Examples:

1. Find the length of the confidence interval 0.275 < 𝑝 < 0.463

109
STEPS SOLUTION
1. Determine the given UCL= 0.463 LCL=
0.275
2. Use the formula for Length of LCI= UCL-LCL
Confidence Interval
3. Substitute the given value, then LCI= 0.463-0.275 LCI=
perform the indicated operation. 0.188
Thus, the length of confidence interval is equal to 0.188

2. Find the length of confidence interval given the following data:

σ = 2.35 n= 250 confidence level= 99%

STEPS SOLUTION
1. Determine the given σ = 2.35, n= 250, confidence level= 99%
2. Determine the 𝑎 in (1- (1 − 𝑎)100% = 99%
a)100% confidence 1 − 𝑎 = 0.99
𝑍𝑎 𝑎 = 0.01
level, then find ⁄2 𝑎 0.01
= = 0.005
2 2
Subtract 0.005 from 0.5 (which is half the area of the
Standard Normal Curve)
0.500 − 0.005 = 0.495
Using the Standard Normal Table,
𝑍𝑎
⁄2 = 2.58
3. Substitute the values in 𝑍𝑎 𝜎
𝐿𝐶𝐼 = 2 ⁄
the formula and
𝐿𝐶𝐼
compute.
√250
𝑍𝑎
𝐿𝐶𝐼 = 2 ⁄ 𝜎
𝐿𝐶𝐼 = 0.7669

Thus, the length of confidence interval is equal to 0.7669

3. Find the length of confidence interval given the following data:

σ = 3.25 n= 17 confidence level= 95%


STEPS SOLUTION
1. Determine the given σ = 3.25, n= 17, confidence level= 95%
2. Find the degree of 𝑑𝑓 = 𝑛 − 1
freedom 𝑑𝑓 𝑑𝑓 = 17 − 1
𝑑𝑓 = 16
3. Determine the 𝑎 in (1- (1 − 𝑎)100% = 95%
a)100% confidence 1 − 𝑎 = 0.95
𝑡𝑎 𝑎 = 0.05
level, then find ⁄2 𝑎 0.05
110
= = 0.025
2 2

Using the t distribution critical values (t-table) at 𝑑𝑓 =


𝑎
16 and = 0.025
2

𝑡𝑎
⁄2 = 2.120
4. Substitute the values in 𝑡𝑎 𝜎
𝐿𝐶𝐼 = 2 ⁄
the formula and
𝐿𝐶𝐼
compute.
17
𝑍𝑎
𝐿𝐶𝐼 = 2 ⁄ 𝜎
for 𝐿𝐶𝐼 = 3.3421
n<30

Thus, the length of confidence interval is equal to 3.3421

LEARNING COMPETENCIES

• Identifies the length of a confidence interval: LC Code: M11/12SP-IIIj-1


• Computes for the length of confidence interval: LC Code: M11/12SP-IIIj-1

DIRECTIONS/INSTRUCTIONS: Read and understand the directions in each activity. If you have any
question, feel free to message your teacher for clarification/assistance.

ACTIVITY 1: WHO IS HE?

Directions: Find out the identity of the man who first introduced the modern concept of confidence
interval into statistical hypothesis testing by identifying the length of the confidence interval and use
the corresponding letter to complete the table provided below. Some letter/s will be left unused.
Show your solutions.

111
Name
Answer 0.145 0.188 0.234 0.249 0.199 0.286 0.188 0.225 0.198 0.237 0.286

ACTIVITY 2: CRACK THE CODE AND OPEN THE LOCK!

Directions: In this Crack the Code puzzle, you are given three digits, some hints are already given
regarding the correctness of these digits. Your challenge is to decode the correct 3 digits code by
computing the length of the confidence interval and then open the lock. Show your solutions.

112
So, are you ready to crack the code?

One number is correct and


2 1 4 well placed

Nothing is correct
7 6 2
Two numbers are correct but
5 3 0 wrongly placed
? ? ?

Find the length of the confidence interval given the following data:
1 5
σ = 3.72 n= 150 confidence level= Fifteen individuals were asked how long they
99% slept at night. The mean time was 7.1 hours
and the standard deviation was 0.78 hour. If
the 90% confidence interval was adopted, find
the length of the confidence interval.

2 6
σ = 1.20 n= 17 confidence level= 90% σ = 5.19 n=530 confidence level= 99%

3 7
σ = 3.25 n= 29 confidence level= 95% Consider a random variable that is normally
distributed with a standard deviation of 5 and a
4 confidence level= 95% sample size of 230. Find the length of
σ = 6.5 n= 35 confidence interval at a confidence level of 95%

Code
Answer 2.4720 1.5673 0.7093

REFLECTION

1. Cite an example where confidence interval is used in real life.


___________________________________________________________________________
___________________________________________________________________________
_______________________________________________________________
113
2. Which do you think is the best confidence interval to use? Why?
___________________________________________________________________________
___________________________________________________________________________
_______________________________________________________________

REFERENCES FOR LEARNERS

Bluman, A. G. (2009). Elementary Statistics: A step by Step Approach (7th Edition). In A. G. Bluman,
Elementary Statistics: A step by Step Approach (7th Edition) (pp. 356-392). New York City:
McGraw-Hill Companies, Inc.

Lisa Sullivan, P. (n.d.). Confidence Intervals. Retrieved from Boston University School of Public
Health: https://sphweb.bumc.bu.edu/otlt/mph-
modules/bs/bs704_confidence_intervals/bs704_confidence_intervals_print.html

Murphy, M. (2017, April 13). Amstat News: The Membership Magazine of the American Statistical
Association. Retrieved from https://magazine.amstat.org/blog/2017/04/13/sih-neyman/

Rene R. Belecina, E. S. (2016). RBS Statistics and Probability (First Edition). In E. S. Rene R. Belecina,
RBS Statistics and Probability (First Edition) (pp. 200-210). 856 Nicanor Reyes
Sr.St.,Sampaloc, Manila: Rex Book Store, Inc (RBSI).

114
ANSWER KEY

Prepared by:

MARY FRANCIA S. RICO, RChE


Writer

115
STATISTICS AND PROBABILITY

Name of Learner: _________________________________ Grade Level: ____________


Section: __________________________________________Date: __________________

LEARNING ACTIVITY SHEET


DETERMINING THE SAMPLE SIZE DETERMINATION USING
THE LENGTH OF CONFIDENCE LEVEL INTERVAL
BACKGROUND
INFORMATION FOR LEARNERS
Two things to remember when we decide on the quality of the sample size we need:
confidence and the narrowness of the interval.
The computing formula in determining sample size is derived from the following formula:
Deriving Formula 1 Margin of Error

𝒁𝒂 𝝈
𝑬= ⁄𝟐

Formula in Determining Minimum Sample Size Needed when Estimating the Population
Mean:

𝒁𝒂
𝒏 ⁄𝟐 𝝈𝟐
𝑬

Note: When determining sample size, we always round up the resulting value to the next
whole number.

Example:
Teacher France wants to conduct a survey about the average number of students in her
school who wants online class instead of distance modular learning. If she plans to use 99%
confidence level, 0.5 as the margin of error, and a standard deviation of 5. How many
sample sizes does she need for the survey?
STEPS SOLUTIONS
Step 1: the Confidence Level=99%; E=0.5; σ = 5
Determine given.

Step 2: Determine the (1 − 𝑎)100% = 99%


confidence coefficient. 1 − 𝑎 = 0.99
116
𝑎 = 0.01
𝑎 0.
= = 0.005
2

Subtract 0.005 from 0.5 (which is half the area of the


Standard Normal Curve)
0.500 − 0.005 = 0.495

Using the Standard Normal Table,

𝑍𝑎
⁄2 = 2.58

Step 3: Substitute the values 𝑍𝑎


⁄ 𝜎
in the formula and
𝑛
compute
𝑛
𝑛
Thus, Teacher France needs a sample size of 666 for the
survey.

Deriving Formula 2 Margin of Error

𝑬 𝒁𝒂⁄𝟐 𝒑̂𝒏 ̂𝒒

Formula in Determining Minimum Sample Size Needed when Estimating the Population
Proportion:

𝟐
𝒏 = 𝒑̂ ̂𝒒 (𝒁𝒂⁄ )𝟐
𝑬 Where: ̂𝒑 = point
estimate for the true population proportion, 𝒑̂ = 𝒙
𝒏
x = number of successes in the sample
̂𝒒 = point estimate for the population proportions of failures, ̂𝒒
= 𝟏 − 𝒑̂

Note: This formula assumes that you have preliminary estimates for 𝑝̂ and 𝑞
̂ . If not, use
𝑝̂ = 0.5 and ̂𝑞 = 0.5

117
Example:
Susan wants to know with 95% confidence the proportion of households who likes to use
Detergent X. A previous survey showed that 42% like to use Detergent X. Susan likes to be
accurate within 2% of the true proportion. What sample size does Susan need?
STEPS SOLUTIONS
Step 1: Determine Confidence Level=95%; E=2% or 0.02; proportion=42%
the given.

Step 2: Determine the With 95% confidence level, a= .05


confidence coefficient. So,
𝑍𝑎
⁄2 = 1.96
Step 3: Determine 𝒑̂ and ̂𝒒. The given proportion is 42% That
is 𝑝̂ = 0.42.
So, 𝑞 ̂ = 1 − ̂𝑝 = 1 − 0.42 = 0.58
Step 4: Substitute the 2
𝑛 = 𝑝̂𝑞̂ (𝑍𝑎⁄ )2
values in the formula and
𝐸
compute 2
1.
𝑛 = (0.42)(0.58) ( )
0.
𝑛 = (0.42)(0.58)(9604)
𝑛 = 2339.53 𝑜𝑟 2340
Thus, Susan needs a sample of 2340 respondents.

LEARNING COMPETENCIES

• Computes for an appropriate sample size using the length of the confidence
interval. LC Code: M11/12SP-IIIj-3
• Solve problems involving sample size determination. LC Code: M11/12SP-
IIIj-4

Directions/Instructions: Read and understand the directions in each activity. If you have any
question, feel free to message your teacher for clarification/assistance.

118
Activity 1: I CAN DO THIS!
Directions: Determine the sample size given the following data.

1. S=10 E= 5 Confidence level= 95%


2. S=3.30 E= 1.03 Confidence level= 99%
3. S= 8 E= 0.25 Confidence level= 90%
4. p̂ = 0.52 E= 0.18 95% confidence
5. p̂ = 0.38 E= 0.08 90% confidence

Activity 2: WHICH WAY TO GO?


Directions: In this maze, the path from the entrance to the exit is hidden by math problems.
Answer all of the problems correctly and it will guide you to the end of the maze. Make a
mistake, and you will be led in the wrong direction. Worse, taking a wrong turn could reveal
a path of correct answers that eventually dead-ends.
START HERE!!

The Principal wants to use the


A medical research team feels
proportion of a population to
determine the sample size needed to confident that a vaccine they have interview regarding
their thoughts developed will cure about 75% of about the new school structure. She 350 the
patients suffering from COVID19 virus. How large should the wants to be able to assert with a
probability 0.95 that her error will be at sample size be for the team to be most 0.05. Similar
polls in the past 98% confident that the sample showed that 65% approved the new
proportion of the cures is within 4% structure. How large a sample does the of the
proportion?
Principal need?

914 10

You are running a political campaign a


A statistic practitioner would like to nd wish to estimate, with 95% confide estimate a
population mean within nce, the population proportion of regi

10 units with 99% confidence given steredndidate. voters Your who estimate will vote
must for be your accur ca

119
that the population standard 4161 ate within 3% of the population propo

deviation is 250. What sample size rtion. Find the minimum sample size

should be used?
needed when no preliminary estimate

is available.

2150 1068

A travel agent wants to estimate the


proportion of vacationers who plan to FINISH travel outside the Phi lippines in
the next 12 months. A random sample of 150 vacationers revealed that 45 had
REFLECTIONplans for foreign travel in that time
505
frame. Suppose (at the 95% confidence
1. What happens to the length of confidence interval when the sample size increases? level) you
need to have a margin of
error no more than 4 percentage
_____________________________________________________________________ points.
How many
vacation_____________________________________________________________________er
s would you have to sample?
_____________________________________________________________________ ______
2. Does increasing sample size increase accuracy or precision? Explain your answer.
_____________________________________________________________________
_____________________________________________________________________
_____________________________________________________________________
______

REFERENCES FOR LEARNERS

Bluman, A. G. (2009). Elementary Statistics: A step by Step Approach (7th Edition). In A. G.


Bluman, Elementary Statistics: A step by Step Approach (7th Edition) (pp. 356-392). New
York City: McGraw-Hill Companies, Inc.
Lisa Sullivan, P. (n.d.). Confidence Intervals. Retrieved from Boston University School of Public
Health: https://sphweb.bumc.bu.edu/otlt/mph-
modules/bs/bs704_confidence_intervals/bs704_confidence_intervals_print.html

120
Rene R. Belecina, E. S. (2016). RBS Statistics and Probability (First Edition). In E. S. Rene R.
Belecina, RBS Statistics and Probability (First Edition) (pp. 200-210). 856 Nicanor Reyes
Sr.St.,Sampaloc, Manila: Rex Book Store, Inc (RBSI).
Statistics Lesson 6.3 Notebook. (2017, May 03). Retrieved from
https://www.hanover.k12.in.us/cms/lib/IN01001361/Centricity/Domain/104/Stats%
206.3.pdf

ANSWER KEY

121
122
Writer
MARY FRANCIA S. RICO, RChE
Prepared by:
Activity 1: I CAN DO THIS!
1. 16
2. 69
3. 15
4. 30
5. 100
Activity 2: WHICH WAY TO GO?

You might also like